Sunteți pe pagina 1din 55

Problem F02.10.

Let T be a linear operator on a finite dimensional complex inner prod-


uct space V such that T ∗ T = T T ∗ . Show that there is an orthonormal basis of V consisting
of eigenvectors of B.

Solution. When T satisfies T ∗ T = T T ∗ , we call T normal.


We prove this by induction on the dimension of the space that T operates on. If T is
operating on a 1-dimensional space, the claim is obvious.
Suppose the claim holds for any normal T operating on an n − 1 dimensional space
(n ≥ 2). By the fundamental theorem of algebra, the characteristic polynomial of T ∗ has a
root λ which is an eigenvalue of T ∗ . Let v be the corresponding non-zero eigenvector (wlog
||v|| = 1). Then v ⊥ = {x ∈ V : (x, v) = 0} has dimension n − 1. Also, if x ∈ v ⊥ , then
(T x, v) = (x, T ∗ v) = λ(x, v) = 0.
Thus v ⊥ is T -invariant. Then the restriction of T to v ⊥ is a normal operator on an n−1 dimen-
sional space. Then by our inductive hypothesis, there is an orthonormal basis {v2 , . . . , vn }
for v ⊥ consisting of eigenvectors of T . Then {v, v2 , . . . , vn } is an orthonormal set with n
elements and is thus a basis for V . It remains to prove that v is also an eigenvector of T and
then we will have the required basis. Since T is normal, so is T ∗ and thus T ∗ − cI for every
c ∈ C. Also, for any normal operator S and any vector x, we see
||Sx||2 = (Sx, Sx) = (x, S ∗ Sx) = (x, SS ∗ x) = (S ∗ x, S ∗ x) = ||S ∗ x||2 .
Then since v is an eigenvector of T ∗ , we have (T ∗ − λI)v = 0. Then
2
0 = ||(T ∗ − λI)v||2 = ||(T ∗ − λI)∗ v||2 = (T − λI)v .

Thus T v = λv so v is also an eigenvector of T . Thus {v, v2 , . . . , vn } is a basis of V consisting


of eigenvectors of T . This completes the induction and the proof.

Problem W02.8. Let T : V → W and S : W → X be linear transformations of real finite


dimensional vector spaces. Prove that
rank(T ) + rank(S) − dim(W ) ≤ rank(S ◦ T ) ≤ max{rank(T ), rank(S)}.
Solution. By the Rank-Nullity Theorem,
rank(T ) + dim(ker(T )) = dim(V ), (1)
rank(S) + dim(ker(S)) = dim(W ), (2)
rank(S ◦ T ) + dim(ker(S ◦ T )) = dim(V ). (3)
Adding the (1), (2) and then subtracting (3) gives
rank(T ) + rank(S) − rank(S ◦ T ) + dim(ker(T )) + dim(ker(S)) − dim(ker(S ◦ T )) = dim(W ).
Let {v1 , . . . , v` } be a basis for ker(T ). Then (S ◦ T )(vi ) = 0 for each i so ker(T ) ⊂ ker(S ◦ T ).
Thus we can extend this to a basis {v1 , . . . , v` , y1 , . . . , yk } for ker(S ◦ T ). Then for each
j = 1, . . . , k, we have
0 = (S ◦ T )(yj ) = S(T (yj )).

1
Hence T (yj ) ∈ ker(S) for each j. Further if a1 , . . . , ak ∈ C are such that

a1 T (y1 ) + · · · + ak T (yk ) = 0,

Then
T (a1 y1 + · · · + ak yk ) = 0
so a1 y1 + · · · + ak yk ∈ ker(T ) so there are b1 , . . . , b` ∈ C such that

a1 y1 + · · · + ak yk = b1 v1 + · · · b` v` =⇒ a1 y1 + · · · + ak yk − b1 v1 − · · · − b` v` = 0.

But these vectors form a basis for ker(S ◦ T ) so in particular, a1 = · · · = ak = 0. Thus


{T (y1 ), . . . , T (yk )} is a linearly independent subset of ker(S) and so dim(ker(S)) ≥ k. Hence

dim(ker(T )) + dim(ker(S)) ≥ dim(ker(S ◦ T ))

and so the equation above yields

rank(T ) + rank(S) − rank(S ◦ T ) ≤ dim(W )

or
rank(T ) + rank(S) − dim(W ) ≤ rank(S ◦ T )
which is the first half of the inequality.
Now suppose that {x1 , . . . , xm } is a basis for im(S ◦ T ). Then there are u1 , . . . , um ∈ V
such that (S ◦ T )(ui ) = xi , i = 1, . . . , m. Thus S(T (ui )) = xi for each i, and so in particular
xi ∈ im(S) for each i and so we have m linearly independent vectors in im(S). This gives
rank(S ◦ T ) ≤ rank(S) ≤ max{rank(S), rank(T )}. This is the second half of the inequality.

Problem W02.10. Let V be a finite dimensional complex inner product space and
f : V → C a linear functional. Show that there exists a vector w ∈ V such that f (v) = (v, w)
for all v ∈ V .

Solution. Let v1 , . . . , vn be an orthonormal basis for V . Given f ∈ V ∗ , put f (vi ) = αi ∈ C.


Next set
w = α1 v1 + · · · + αn vn .
For any v ∈ V , there are β1 , . . . , βn ∈ C such that

v = β1 v1 + · · · + βn vn .

Then
f (v) = β1 f (v1 ) + · · · + βn f (vn ) = β1 α1 + · · · + βn αn
and !
n
X n
X n X
X n n X
X n
(v, w) = βi vi , αj vj = (βi vi , αj vj ) = βi αj (vi , vj ).
i=1 j=1 i=1 j=1 i=1 j=1

Thus by orthonormality,
n
X
(v, w) = βi αi = f (v).
i=1

2
Since v was arbitrary, f (v) = (v, w) for all v ∈ V .

Problem W02.11. Let V be a finite dimensional complex inner product space and
T : V → V a linear transformation. Prove that there exists an orthonormal ordered basis
for V such that the matrix representation of T in this basis is upper triangular.

Solution. We prove this by induction on the dimension of the space T acts upon. If T is
acting on a 1-dimensional space, the claim is obvious.
Suppose the claim holds for linear maps acting on n−1 dimensional spaces. Let dim(V ) =
n. By the fundamental theorem of algebra, there is an eigenvalue λ ∈ C and corresponding
non-zero eigenvector 0 6= v1 ∈ V of T ∗ ; wlog ||v1 || = 1. Then v1⊥ = {x ∈ V : (x, v1 ) = 0} is
an n − 1-dimensional space. Also for x ∈ v1⊥ , we have

(T (x), v1 ) = (x, T ∗ (v1 )) = (x, λv1 ) = λ(x, v1 ) = 0.



Hence v1⊥ is T -invariant. Thus T v⊥ is an operator acting on an n − 1-dimensional space.
1
By our inductive hypothesis, there is an orthonormal basis {v2 , . . . , vn } for v ⊥ such that
the matrix of T is upper-triangular with respectPn to this basis. Then {v1 , v2 , . . . , vn } is an
orthonormal
Pn basis for V . Further, T (v1 ) = j=1 a1j vj for some a1j ∈ C and by assumption
T (vi ) = j=i aij xj . Thus the matrix of T with respect to this basis is
 
a11 a12 a13 ··· a1n
 0 a22 a23
 ··· a2n 

A= 0
 0 a33 ··· a3n 
.
 .. .. .. 
 . . . 
0 0 0 ··· ann

Problem S03.8. Let V be an n-dimensional complex vector space and T : V → V a


linear operator. Suppose that the characteristic polynomial of T has n distinct roots. Show
that there is a basis B of V such that the matrix representation of T in the basis B is diagonal.

Solution. Since each root of the characteristic polynomial (and thus each eigenvalue of T )
is distinct and since eigenvectors corresponding to different eigenvalues are linearly indepen-
dent, each eigenspace Eλ is a one-dimensional T -invariant subspace. Let λ1 , . . . , λn be the
distinct eigenvalues of T with corresponding eigenvectors v1 , . . . , vn . We know that eigen-
vectors corresponding to distinct eigenvalues are linearly independent, thus Eλi ∩ Eλj = {0}
whenever i 6= j. Further, since we have n-linearly independent vectors, {v1 , . . . , vn } is a basis
for V . The matrix of T with respect to this basis is
 
λ1
 λ2 0 
..
 
[T ] = 
 . ,

 . ..

 0 
λn

3
since T (vi ) = λi vi , i = 1, . . . , n.

Problem S03.9. Let A ∈ M3 (R) satisfy det(A) = 1 and At A = I = AAt where I is the
identity matrix. Prove that the characteristic polynomial of A has 1 as a root.

Solution. Clearly the characteristic polynomial of A has a real root since it has odd order.
Let λ be a real root of the characteristic polynomial. Then λ is an eigenvalue of A. Suppose
0 6= v ∈ R3 is a normalized eivengector corresponding to λ. Then

λ2 = λ2 (v, v) = (λv, λv) = (Av, Av) = (v, At Av) = (v, v) = 1.

Thus λ = ±1. If λ = 1, then we are done. If λ = −1, suppose µ, ν ∈ C are the other
eigenvalues of A. Then −1 = − det(A) = −µνλ = µν. If µ, ν are not real, they must be a
conjugate pair since A is real. But this is impossible, because then µν ≥ 0. Thus both µ, ν
are real. By the same reasoning as above, µ, ν = ±1. Then µν = −1 forces µ = 1, ν = −1
(or vice versa). Thus A has 1 as an eigenvalue and so the characteristic polynomial of A has
1 as a root.

Problem S03.10. Let V be a finite dimensional real inner product space and T : V → V
a hermitian linear operator. Suppose the matrix representation of T 2 in the standard basis
has trace zero. Prove that T is the zero operator.

Solution. Let dim(V ) = n and let A be the matrix of T in the standard basis. Since
T is hermitian, so is A and thus by the spectral theorem, there is an orthonormal basis
{v1 , . . . , vn } for Rn consisting of eigenvectors of A. Let λ1 , . . . , λn ∈ R be the corresponding
eigenvalues (repeats are allowed and the eigenvalues are real since A is hermitian). Then

Avi = λi vi =⇒ A2 vi = λi Avi = λ2i vi .

Thus (λ2i , vi ) is an eigenpair for A2 which is the matrix of T 2 . We are given that the trace
of the matrix is zero, but the trace is the sum of the eigenvalues. Hence
n
X
λ2i = 0 =⇒ λ1 = · · · = λn = 0;
i=1

again this holds since all eigenvalues of a hermitian operator are real. Then Avi = 0, i =
1, . . . , n. But this means A sends a basis for Rn to zero. This is only possible if A is the zero
matrix. Thus T is the zero transformation.

Problem F03.9. Consider a 3×3 real symmetric matrix with determinant 6. Assume that
(1, 2, 3) and (0, 3, −2) are eigenvectors with corresponding eigenvalues 1 and 2 respectively.

(a) Give an eigenvector of the form (1, x, y) which is linearly independent from the two
vectors above.

(b) What is the eigenvalue of this eigenvector?

4
Solution. We answer the questions in the reverse order. The product of the eigenvalues
equals the determinant, so the third eigenvalue is 3. This answers (b).
By the spectral theorem, the eigenspaces corresponding to distinct eigenvalues will be
orthogonal. Here all eigenvalues are distinct. Since the first two eigenvectors span a two
dimensional space, any vector orthogonal to both will necessarily be a third eigenvector.
Taking the cross product of the two vectors gives a vector which is orthogonal to both. We
see
ı̂ ̂ k̂

(1, 2, 3) × (0, 3, −2) = 1 2 3 = (−13, 2, 3).
0 3 −2

Then, v = (1, −2/13, 3/13) is an eigenvector of the desired form. This answers (a).

Problem F03.10.

(a) Take t ∈ R such that t is not an integer multiple of π. Prove that if


 
cos(t) sin(t)
A=
− sin(t) cos(t)

then there is no invertible real matrix B such that BAB −1 is diagonal.

(b) Do the same for  


1 λ
A=
0 1
where λ ∈ R − {0}.

Solution.

(a) A doesn’t have any real eigenvalues so it cannot be diagonalizable in M2 (R). Indeed,
any eigenvalue λ of A would need to satisfy

(cos(t) − λ)2 + sin2 (t) = 0 =⇒ cos(t) − λ = ±i sin(t) =⇒ λ = cos(t) ± i sin(t)

which is not real since t is not an integer multiple of π.

(b) The only eigenvalue of A is 1 and it has algebraic multiplicity 2. However, the only
eigenvalue corresponding to this eigenvalue (up to scaling) is v = (1, 0). Thus the
geometric multiplicity of the eigenvalue is 1. Since A has an eigenvalues whose algebraic
and geometric multiplicities are unequal, A is not diagonalizable.

Problem S04.7. Let V be a finite dimensional real vector space and U, W ⊂ V be


subspaces of V . Show both of the following:

(a) U 0 ∩ W 0 = (U + W )0

(b) (U ∩ W )0 = U 0 + W 0

Solution.

5
(a) Let f ∈ U 0 ∩ W 0 . Then f ∈ U 0 and f ∈ W 0 . Take x ∈ U + W . Then x = u + w for
some u ∈ U, w ∈ W . We see

f (x) = f (u + w) = f (u) + f (w) = 0 + 0 = 0.

Thus f ∈ (U + W )0 .
Now take f ∈ (U + W )0 . For any u ∈ U , we have u ∈ U + W . Then f (u) = 0. Hence,
since u was arbitrary, f ∈ U 0 . Similarly, f (w) = 0 for all w ∈ W so f ∈ W 0 . Thus
f ∈ U 0 ∩ W 0.
We conclude that U 0 ∩ W 0 = (U + W )0 .

(b) Let f ∈ (U ∩ W )0 . For any Z ⊂ V , define fZ ∈ V ∗ by fZ (v) = f (v), v ∈ Z,


fZ (v) = 0, v ∈ V − Z. Then
f = fU + fV −U .
Clearly fV −U ∈ U 0 . We must show fU ∈ W 0 . Let w ∈ W . If w ∈ U , then fV −U (w) = 0,
and f (w) = 0 since w ∈ U ∩W . Thus 0 = f (w) = fU (w)+fV −U (w) = fU (w). Otherwise
w 6∈ U , in which case fU (w) = 0 by definition. Hence, fU (w) = 0 for all w ∈ W and so
fU ∈ W 0 . Then
f = fU + fV −U ∈ U 0 + W 0 .

Take f ∈ U 0 + W 0 . Then f = f1 + f2 , f1 ∈ U 0 , f2 ∈ W 0 . For any v ∈ U ∩ W , we have


v ∈ U and v ∈ W . Then

f (v) = f1 (v) + f2 (v) = 0 + 0 = 0.

Hence f (v) = 0 for all v ∈ U ∩ W so f ∈ (U ∩ W )0 .


We conclude (U ∩ W )0 = U 0 + W 0 .

Problem S04.9. Let V be a finite dimensional real inner product space and T : V → V a
linear operator. Show the following are equivalent:

(a) (T x, T y) = (x, y) for all x, y ∈ V ,

(b) ||T x|| = ||x|| for all x ∈ V ,

(c) T ∗ T = I, where T ∗ is the adjoint of T and I : V → V is the identity map,

(d) T T ∗ = I.

Solution. (a) =⇒ (b) by taking x = y.


Assume (b) is true. Then for any x, y ∈ V ,

(x, x) + 2(x, y) + (y, y) = (x + y, x + y)


= (T x + T y, T x + T y)
= (T x, T x) + 2(T x, T y) + (T y, T y)
= (x, x) + 2(T x, T y) + (y, y).

6
Thus 2(x, y) = 2(T x, T y) and so (b) =⇒ (a).
Assume (a), (b) are true. Consider, for any v ∈ V ,

((T ∗ T − I)v, (T ∗ T − I)v) = (T ∗ T v − v, T ∗ T v − v)


= (T ∗ T v, T ∗ T v) − (T ∗ T v, v) − (v, T ∗ T v) + (v, v)
= (T ∗ T v, T ∗ T v) − (T v, T v) − (T v, T v) + (v, v)
= (T ∗ T v, T ∗ T v) − (v, v) − (v, v) + (v, v) [by (a)]
= (T ∗ T v, T ∗ T v) − (v, v)
= (T v, T T ∗ T v) − (v, v)
= (v, T ∗ T v) − (v, v) [by (a)]
= (T v, T v) − (v, v) = 0 [by (b)].

Thus (T ∗ T − I)v = 0 for all v ∈ V so T ∗ T = I. Thus (b) =⇒ (c).


Assume (c) is true. Then for any v ∈ V ,

(v, v) = (v, Iv) = (v, T ∗ T v) = (T v, T v).

Hence (c) =⇒ (b).


Thus far we have that (a),(b),(c) are equivalent. Assume these hold.Then (a) implies

(T T ∗ x, T T ∗ y) = (T ∗ x, T ∗ y) = (T T ∗ x, y) =⇒ (T T ∗ x, (T T ∗ −I)y) = 0

for all x, y ∈ V . From (c) we see that T, T ∗ are bijective, thus so is T T ∗ . Hence (T T ∗ − I)y
is orthogonal to all of V so (T T ∗ − I)y = 0. However y was arbitrary, so this implies that
T T ∗ − I = 0 so T T ∗ = I. Thus (a),(b),(c) imply (d).
Assume (d) is true. Then for any x, y ∈ V , we have

(x, y) = (x, Iy) = (x, T T ∗ y) = (T ∗ x, T ∗ x).

Then

(T ∗ T x, T ∗ T y) = (T x, T y) = (T ∗ T x, y) =⇒ (T ∗ T x, (T ∗ T − I)y) = 0.

This implies (c) (and thus (a),(b)) by the same reasoning as above.

Problem F04.10. Let T : Rn → Rn be a linear transformation and for λ ∈ C, define the


subspace V (λ) = {v ∈ V : (T − λI)n v = 0 for some n ≥ 1}. This is called the generalized
eigenspace of λ.

1. Prove there for each λ ∈ C there is a fixed N ∈ N such that V (λ) = ker (T − λI)N .

2. Prove that if λ 6= µ then V (λ) ∩ V (µ) = {0}. Hint: use the fact that
T − λI T − µI
I= + .
µ−λ λ−µ

Solution.

7
1. Let v1 , . . . , vk be a basis for V (λ). For i = 1, . . . , k, define Ni ∈ N to be equal to
the least n ∈ N such that (T − λI)n vi = 0. Take N = max{N1 , . . . , Nk }. Then
(T − λI)N vi = 0 for all i = 1, . . . , k. Since we can build any vector in V (λ) from a
linear combination of v1 , . . . , vk , we see V (λ) = ker (T − λI)N .
2. By part 1., there are N1 , N2 ∈ N such that
V (λ) = ker (T − λI)N1 V (µ) = ker (T − µI)N2 .
 
and
Take M = max{N1 , N2 }. We see that
 2M
2M T − λI T − µI
I=I = +
µ−λ λ−µ
Since (T − λI) and (T − µI) commute with each other, by the binomial theorem, if
we expand the right hand side above, each summand will have a factor of (T − λI) or
(T − µI) which has a power greater than or equal to M . Thus if v ∈ V (λ) ∩ V (µ), then
 2M
T − λI T − µI
v = Iv = + v = 0.
µ−λ λ−µ
Hence V (λ) ∩ V (µ) = {0}.
Problem S05.1. Given n ≥ 1, let tr : Mn (C) → C denote the trace operator:
n
X
tr(A) = Akk , A ∈ Mn (C).
k=1

(a) Determine a basis for the kernel of tr.


(b) For X ∈ Mn (C), show that tr(X) = 0 if and only if there are matrices A1 , . . . , Am ,
B1 , . . . , Bm ∈ Mn (C) such that
m
X
X= Aj Bj − Bj Aj .
j=1

Solution.
(a) Since dim(Mn (C)) = n2 and since the range of tr is C which has dimension 1 as a vector
space over itself, we know from the Rank-Nullity theorem that dim ker(tr) = n2 − 1.
Thus to find a basis for the kernel, it is sufficient to find n2 − 1 linearly independent
matrices on which the trace operator vanishes. For i = 1, . . . , n, j = 1, . . . n,, define
Eij ∈ Mn (C) be such that the entry in the ith row and j th column is 1 and the other
entries are 0. Then the matrices Eij , i 6= j are clearly linearly independent and have 0
trace. This constitutes n2 − n linearly independent matrices with 0 trace. For n − 1
more, consider Fk = E11 −Ekk , k = 2, . . . , n. These are also linearly independent. Thus
the collection
{Eij : i, j ∈ {1, . . . , n}, i 6= j} ∪ {Fk : k = 2, . . . , n}
forms a basis for the kernel of tr.

8
(b) It is well-known that tr(AB) = tr(BA), A, B ∈ Mn (C). Thus if X has the specified
form,
m
X m
X
tr(X) = tr(Aj Bj ) − tr(Bj Aj ) = tr(Aj Bj ) − tr(Aj Bj ) = 0.
j=1 j=1

Conversely, if X has trace 0, we must have some complex numbers αij , 1 ≤ i, j ≤ n, i 6=


j and βk , k = 2, . . . , n such that
n
X n
X
X= αij Eij + βk F k .
i,j=1,i6=j k=2

Further, we see that


Eij = Eii Eij and 0 = Eij Eii .
Thus
αij Eij = Eii (αij Eij ) − (αij Eij )Eii , i, j = 1, . . . , n, i 6= j.
Further
Fk = Ek1 E1k − E1k Ek1 , k = 2, . . . , n.
Thus
n
X n
X
X= Eii (αij Eij ) − (αij Eij )Eii + (βk Ek1 )E1k − E1k (βk Ek1 ).
i,j=1,i6=j k=2

This is the desired form up to renaming matrices and indices.

Problem S05.2. Let V be a finite-dimensional vector space and let V ∗ denote the dual
space of V . For a set W ⊂ V , define

W ⊥ = {f ∈ V ∗ : f (w) = 0 for all w ∈ W } ⊂ V ∗

and for a set U ⊂ V ∗ , define



U = {v ∈ V : f (v) = 0 for all f ∈ U } ⊂ V.

(a) Show that for any W ⊂ V , ⊥ (W ⊥ ) = Span (W ) .

(b) Let W be a subspace of V . Give an explicit isomorphism bewteen (V /W )∗ and W ⊥ .


Show that it is an isomorphism.

Solution.

(a) Let W ⊂ V . Take x ∈ Span (W ). Then there are scalars α1 , . . . , αn and vectors
w1 , . . . , wn ∈ W such that
x = α1 w1 + · · · + αn wn .
Then for any f ∈ W ⊥ we have

f (x) = α1 f (w1 ) + · · · + αn f (wn ) = α1 · 0 + · · · + αn · 0 = 0.

9
Since f was arbitrary, f (x) = 0 for all f ∈ W ⊥ . Hence by definition x ∈⊥ (W ⊥ ). Thus
Span (W ) ⊂⊥ (W ⊥ ).
Now take x ∈ W , then for all f ∈ W ⊥ , we have f (x) = 0. Hence, x ∈⊥ (W ⊥ ). However,
Span (W ) is the smallest vector space containing W . Thus ⊥ (W ⊥ ) ⊂ Span (W ).
Thus ⊥ (W ⊥ ) = Span (W ) .

(b) Define φ : (V /W )∗ → W ⊥ by

φ(f ) = gf , f ∈ (V /W )∗

where gf is the functional which sends x to f (x + W ). Then for any w ∈ W , we have

[φ(f )](w) = gf (w) = f (w + W ) = f (0) = 0.

Thus φ(f ) is indeed in W ⊥ when f ∈ (V /W )∗ . Further, if φ(f ) = 0, then

[φ(f )](x) = 0, =⇒ f (x + W ) = 0 for all x ∈ V.

Hence f is the zero functional. Thus φ(f ) = 0 implies f = 0 which tells us that φ is
injective. Then since dim((V /W )∗ ) = dim(V /W ) = dim(V ) − dim(W ) = dim(W ⊥ ),
the injectivity of φ implies bijectivity, so φ is an isomorphism.

Problem S05.3. Let A be a Hermitian-symmetric n × n complex matrix. Show that if


(Av, v) ≥ 0 for all v ∈ Cn then there exists and n × n matrix T such that A = T ∗ T .

Solution. By the spectral theorem, there is an orthonormal basis {v1 , . . . , vn } of Cn con-


sisting of eigenvectors of A. Let λ1 . . . , λn be the corresponding eigenvalues. Since A is
Hermitian, all λi are real. Further, for each i,

0 ≤ (Avi , vi ) = (λi vi , vi ) = λi (vi , vi ) = λi .



Thus we can define a matrix T such that T xi = λi xi (recall that a linear operator is
√ √
uniquely identified by how it treats basis vectors). Then T ∗ xi = λi xi = λi xi since T is
clearly diagonal with respect to this basis. Then
p p  p p 
∗ ∗
Axi = λi xi = λi λi xi = λi T xi = T λi xi = T ∗ (T xi ) = (T ∗ T )xi .

Then since A and T ∗ T agree on basis elements, they are the same matrix.

Problem S05.4. We say that I ⊂ Mn (C) is a two-sided ideal in Mn (C) if

(i) for all A, B ∈ I, A + B ∈ I,

(ii) for all A ∈ I and B ∈ Mn (C), AB and BA are in I.

10
Show that the only two-sided ideals in Mn (C) are {0} and Mn (C) itself.

Solution. Let I be a two sided ideal in Mn (C). Suppose there is a non-zero matrix A ∈ I.
Then by multiplying A by elementary matrices we can transform A → A which has a non-
zero entry in its first row and column and is still in I. Next, by multiplying on the left and
right by a matrix B such that (B)11 = 1 and (B)ij = 0 otherwise. We arrive at a matrix
A∗ ∈ I which is zero everywhere except for a nonzero entry in the first row and column.
Again, multiplying by an elementary matrix, we can reduce A∗ to a matrix A11 ∈ I which
has a 1 in the first row and first column and zeroes elsewhere. Performing similar steps,
we can create matrices Aii ∈ I which have a 1 in the ith row and ith column and zeroes
elsewhere. Adding all these matrices together, we see have the identity matrix I ∈ I. Then
for any C ∈ Mn (C), we have CI = C ∈ I. Hence I = Mn (C). Thus the only two-sided
ideals in Mn (C) are {0} and Mn (C).

Problem F05.6.
(a) Prove that if P is a real-coefficient polynomial and A a real-symmetric matrix, then λ
is an eigenvalue of A if and only if P (λ) is an eigenvalue of P (A).
(b) Use (a) to prove that if A is real symmetric, then A2 is non-negative definite.
(c) Check part (b) by verifying directly that det(A2 ) and trace(A2 ) are non-negative when
A is real-symetric.
Solution.
(a) Suppose λ is an eigenvalue of A. Then there is an eigenvector v 6= 0 such that Av = λv
Then
A2 v = A(Av) = A(λv) = λAv = λ2 v.
Similarly, Ak v = λk v for k = 3, 4, 5, . . . . Let P be the polynomial

P (x) = a0 + a1 x + · · · + am xm .

Then

P (A)v = a0 Iv + a1 Av + · · · + am Am v
= a0 v + a1 λv + · · · + am λm v
= (a0 + a1 λ + · · · + am λm )v = P (λ)v.

So P (λ) is an eigenvalue of P (A) (note, we didn’t need the assumption that A is


real-symmetric for this direction).
By the spectral theorem, A is (orthogonally) similar to a diagonal matrix D. Say
A = U DU ∗ where U is orthogonal. Then

P (A) = a0 I + a1 U DU ∗ + · · · + am (U DU ∗ )m
= a0 U U ∗ + a1 U DU ∗ + · · · + am U Dm U ∗
= U (a0 I + a1 D + · · · + am Dm )U ∗ = U P (D)U ∗ .

11
Thus P (A) is similar to P (D). Now if P (λ) is an eigenvalue of P (A) then it is also
an eigenvalue of P (D). But P (D) is diagonal so P (λ) lies on the diagonal of P (D)
and so λ lies on the diagonal of D and thus is an eigenvalue of D. Then λ is also an
eigenvalue of A since A is similar to D (note, this argument also didn’t require that A
is symmetric, just that A is diagonalizable).

(b) If A is n × n, then there is an orthonormal basis for Rn consisting of eigenvectors


{v1 , . . . , vn } of A. Suppose the corresponding eigenvalues are λ1 , . . . , λn (possibly with
repititions). All these eigenvalues are real since A is symmetric. Also, λ21 , . . . , λ2n are
eigenvalues of A2 corresponding to the same eigenvalues. Finally, for any x ∈ Rn ,
x = α1 v1 + · · · + αn vn for some α1 , . . . , αn ∈ R. Thus gives
n
X
t 2 t
x A x = (α1 v1 + · · · + αn vn ) (α1 λ21 v1 + ··· + αn λ2n vn ) = αi2 λ2i , by orthogonality.
i=1

this last expression is clearly non-negative since all λi and αi are real. Thus A2 is
non-negative definite.

(c) We see
det(A2 ) = det(AA) = det(A)det(A) = (det(A))2 ≥ 0.
Also, the entries of A2 are
n
X
2
(A )ij = aik akj .
k=1

Thus the diagonal entries are


n
X n
X
(A2 )ii = aik aki = a2ik , by symmetry.
k=1 k=1

Thus all diagonal entries of A2 are non-negative and so the trace is non-negative.

Problem F05.9. Suppose U, W are subspaces of a finite-dimensional vector space V .

(a) Show that dim(U ∩ W ) = dim(U ) + dim(W ) − dim(Span (U, W )).

(b) Let n = dim(V ). Use part (a) to show that if k < n then an intersection of k subspaces
of dimension n − 1 always has dimension at least n − k.

Solution.

(a) This is called the dimension formula and is proven as follows.


Let {x1 , . . . , xk } be a basis for U ∩ W . Extend this separately to a basis

{x1 , . . . , xk , u1 , . . . , u` } of U and
{x1 , . . . , xk , w1 , . . . , wm } of W.

12
Then dim(U ∩ W ) = k, dim(U ) = k + ` and dim(W ) = k + m. So it remains to prove
that dim(Span (U, W )) = k + ` + m; the result will follow. To do this, we just throw
all the vectors together and if there is any justice in the world

{x1 , . . . , xk , u1 , . . . , u` , w1 , . . . , wm }

will form a basis for Span (U, W ). Take any y ∈ Span (U, W ). Then y can be built
from vectors in U and W . But each of those vectors can be built by basis vectors of U
and W and so y can be built using the basis vectors of U and W . That is

{x1 , . . . , xk , u1 , . . . , u` }∪{x1 , . . . , xk , w1 , . . . , wm } = {x1 , . . . , xk , u1 , . . . , u` , w1 , . . . , wm }

forms a spanning set for Span (U, W ). Take scalars α1 , . . . , αk , β1 , . . . , β` , γ1 , . . . , γm


such that

α1 x1 + · · · + αk xk + β1 u1 + · · · + β` u` + γ1 w1 + · · · + γm wm = 0.
| {z } | {z } | {z }
..=x ..=u ..=w

Then w = −x − u ∈ U . Also, it is clear w ∈ W . Then w ∈ U ∩ W . Hence there are


scalars µ1 , . . . , µk such that

w = µ 1 x1 + · · · + µ k xk .

Then
γ1 w1 + · · · + γm wm − µ1 x1 − · · · − µk xk = 0.
But these vectors form a basis for W and thus a linealry independent set. Thus
γ1 = · · · = γm = 0 (and the same for µi but these won’t matter). Then w = 0 so
x + u = 0. But the vectors comprising x and u form a basis for U , thus α1 = · · · =
αk = β1 = · · · = β` = 0. Thus

{x1 , . . . , xk , u1 , . . . , u` , w1 , . . . , wm }

is a linearly independent set.


From this we see that there is a basis for Span (U, W ) which has k + ` + m elements
so dim(Span (U, W )) = k + ` + m and the proof is completed.
(b) We prove the claim by induction on k. If k = 1, the result is trivial.
Suppose the result holds from some k ≥ 1. Let V1 , . . . , Vk , Vk+1 be subspaces of V of
dimension n − 1. Then

dim ∩k+1 k
= dim (Vk+1 )+dim ∩ki=1 Vk −dim Span Vk+1 , ∩ki=1 Vi ,
   
i=1 Vi = dim Vk+1 ∩ ∩i=1 Vi

by the dimensionality formula. But Span Vk+1 , ∩ki=1 Vi has dimension at most n, Vk+1
has dimension n − 1 and by our inductive hypothesis, ∩ki=1 Vi has dimension at least
n − k. Then

dim ∩k+1

i=1 Vi ≥ n − 1 + n − k − n = n − k − 1 = n − (k + 1).

Thus the claim holds for k + 1. This completes the proof.

13
Problem F05.10.

(a) For n = 2, 3, 4, . . . , is there an n × n matrix A with An−1 6= 0 and An = 0?

(b) Is there an n × n upper triangular matrix A with An 6= 0 and An+1 = 0?

Solution.

(a) Yes. Let A be the matrix which Ai,i+1 = 1 for i = 1, . . . , n − 1 and Aij = 0 otherwise.
That is, 1 on the first superdiagonal and 0 elsewhere.

(b) No. Suppose An+1 = 0. Suppose that λ is an eigenvalue of A with nonzero eigenvector
v. Then

Av = λv =⇒ A2 v = λAv = λ2 v =⇒ ··· =⇒ An+1 v = λn Av = λn+1 v.

But An+1 = 0 so λn+1 v = 0. Then v 6= 0 implies that λn+1 = 0 which gives λ = 0.


Thus all eigenvalues of A are zero. Then the minimal polynomial of A has only zero
as a root and thus mA (x) = xk for some k ∈ N. However, the degree of mA is at most
n. So
mA (A) = 0 =⇒ Ak = 0, for some k ≤ n =⇒ An = 0.
[Note: the assumption that A is upper triangular is unnecessary.]

Problem S06.7. Prove that if a, λ ∈ C with a 6= 0, then


 
1 a 0
T = 0 1 a
0 0 λ

is not diagonalizable.

Solution. A matrix is diagonalizable if and only if the geometric multiplicity of each


eigenvalue is equal to the algebraic multiplicity of the eigenvalue. Here 1 is an eigenvalue of
T of algebraic multiplicity 2 but
    
0 a 0 v1 0
(T − I)v = 0 =⇒  0 0 a   v2 = 0 .
 
0 0 λ−1 v3 0

Then v2 = v3 = 0 and so v = (1, 0, 0)t spans the eigenspace corresponding to the eigenvalue
1. Hence the geometric multiplicity of this eigenvalue is only 1. Hence the matrix is not
diagonalizable.

Problem S06.8. A linear transformation T is called orthogonal if it is non-singular and


T t = T −1 . Prove that if T : R2n+1 → R2n+1 is orthogonal, then there is v ∈ R2n+1 such that
T (v) = v or T (v) = −v.

14
Solution. The characteristic polynomial pT (t) is an odd degree polynomial over R so there
is a real root λ. Then there is a nonzero vector v ∈ R2n+1 such that T (v) = λv so
λ2 (v, v) = (λv, λv) = (T (v), T (v)) = (v, T t T (v)) = (v, Iv) = (v, v).
Since (v, v) 6= 0, this gives λ2 = 1 so λ = ±1 and hence T (v) = ±v.

Problem S06.9. Let S be a real symmetric matrix.


(a) Prove that all eigenvalues of S are real.
(b) State and prove the spectral theorem.
Solution.
(a) Let λ ∈ C be an eigenvalue of S with nonzero eigenvector v.
λ(v, v) = (λv, v) = (Sv, v) = (v, Sv) = (v, λv) = λ(v, v).
Hence since v 6= 0, we have λ = λ so λ ∈ R.
(b) Spectral Theorem. Let S be a symmetric matrix in Mn (F) where F = R or F = C.
Then there is an orthonormal basis for Fn consisting of eigenvectors of S. In particular,
S is orthogonally diagonalizable.
Proof. We prove this by induction on n where n is the dimension of the space that S
operates on.
For n = 1, the statement is obvious since the operator S simply scales vectors.
Assume that if a symmetric matrix operates on an (n − 1)-dimensional space, then the
statement holds. If S is an n × n matrix, then it operates on Fn . By fundamental
theorem of algebra, there is an eigenvalue λ of S and by the above proof, λ ∈ R. Let
0 6= v ∈ Fn be a corresponding eigenvector; without loss of generality, ||v|| = 1. Then
v ⊥ = {x ∈ Fn : (x, v) = 0} is an (n − 1)-dimensional subspace of Fn . Also, if x ∈ v ⊥ ,
then
(Sx, v) = (x, Sv) = (x, λv) = λ(x, v) = 0.

Hence v ⊥ is an S-invariant subspace. Thus S v⊥ is a symmetric matrix operating on
an (n − 1)-dimensional space. By inductive hypothesis, there is an orthonormal basis
{x1 , . . . , xn−1 } of v ⊥ which consists of eigenvectors of S v⊥ and thus eigenvectors of
S. Then {x1 , . . . , xn−1 , v} is an orthonormal set in Fn (and thus a basis for Fn ) which
consists of eigenvectors of S. In particular, if λ1 , . . . , λn−1 , λ are the corresponding
eigenvalues (there can be repetitions) and we put P = [x1 · · · xn−1 v], then
 
λ1

 λ2 

SP = P 
 ...  ..
 = P D,
 
 λn−1 
λ
so S = P DP −1 and since {x1 , . . . , xn−1 , v} is an orthonormal set P −1 = P t so S =
P DP t and thus S is orthogonally diagonalizable.

15
Problem S06.10. Let Y be an arbitrary set of commuting matrices in Mn (C). Prove that
there exists a non-zero vector v ∈ Cn which is a common eigenvector of all matrices in Y .

Solution. Let A ∈ Y . Then by the fundamental theorem of algebra, A has an eigenvalue


λ. Let Eλ = ker(A − λI). Let 0 6= x ∈ Eλ . Then for arbitrary B ∈ Y ,

Ax = λx =⇒ BAx = B(λx) =⇒ A(Bx) = λ(Bx).

Hence
Bx ∈ Eλ . Thus Eλ is B invariant. Hence by the fundamental theorem of algebra,
B E has an eigenvalue and thus an eigenvector v 6= 0. This v is a simultaneous eigenvector

λ
of A and B. Since B was arbitrary, v is a simultaneuous eigenvector of all matrices in Y .

Problem W06.7. Let V be a complex inner product space. State and prove the Cauchy-
Schwarz inequality for V .

Solution.
Cauchy-Schwarz Inequality. Let V be an inner product space over C and v, w ∈ V .
Then
|(v, w)| ≤ ||v|| ||w||
with equality if and only if v, w are linearly dependent.

Proof. If w = 0, the inequality is trivially satisfied (it is actually equality and v, w are
linearly dependent so all statements hold). Assume w 6= 0. For any c ∈ C, we have

0 ≤ (v − cw, v − cw) = (v, v) − c(v, w) − c(v, w) + |c|2 (w, w).


(v,w)
Putting c = (w,w)
(possible since w 6= 0) we see

|(v, w)|2 |(v, w)|2 |(v, w)|2 (w, w) |(v, w)|2


0 ≤ (v, v) − − + =⇒ ≤ (v, v).
(w, w) (w, w) (w, w)2 (w, w)

Multiplying by (w, w), we see

|(v, w)|2 ≤ (v, v)(w, w) = ||v||2 + ||w||2 =⇒ |(v, w)| ≤ ||v|| ||w|| .

We also notice that equality holds if and only if

(v − cw, v − cw) = 0

which holds if and only if v = cw.

Problem W06.8. Let T : V → W be a linear transformation of finite dimensional innter


product spaces. Show that there exists a unique linear transform T t : W → V such that

(T v, w)W = (v, T t w)V for all v ∈ V, w ∈ W.

16
Solution. We prove uniqueness first. Suppose there are linear maps R, S : W → V both
satisfying the condition. Then for all v ∈ V, w ∈ W ,
(T v, w)W = (v, Rw)V = (v, Sw)V =⇒ (v, (R − S)w)V = 0.
Thus (R − S)w = 0 since it is orthogonal to all of V . However, w was arbitrary so this
implies R = S.
For existence, let {v1 , . . . , vn } and {w1 , . . . , wm } be orthonormal bases of V and W re-
spectively. If T has matrix A with respect to these bases, define T t to have matrix At with
respect to these basis. Then T t satisfies the conditon.

Problem W06.9. Let A ∈ M3 (R) be invertible and satisfy A = At and det(A) = 1. Prove
that 1 is an eigenvalue of A.

Solution. The conclusion isn’t actually true. The matrix


1 
2
0 0
A =  0 31 0
0 0 6
is symmetric and has determinant 1 but doesn’t have 1 as an eigenvalue.
It is likely that we were supposed to assume that At = A−1 rather than At = A. In this
case, see S03.9.

Problem S07.2. Let U, V, W be n-dimensional vector spaces and T : U → V, S : V → W


be linear transformations. Prove that is S ◦ T : U → W is invertible, then both T, S are
invertible.

Solution. First suppose that T is not invertible. Then the kernel of T is nontrivial, so there
is a non-zero vector u ∈ U such that T(u)= 0. But then S ◦ T (u) = 0, so the kernel of S ◦ T
is nontrivial and so S ◦ T is not invertible; a contradiction. Thus T is invertible.
Now assume that S is not invertible. Then the kernel of S is nontrivial so there is a non-
zero vector v ∈ V such that S(v) = 0. However, T is invertible, and thus surjective so there
is u ∈ U such that T (u) = v (and u 6= 0 since v 6= 0). Then S ◦ T (u) = S(T (u)) = S(v) = 0,
so the kernel of S ◦ T is nontrivial and S ◦ T is not invertible; a contradiction. Hence S is
invertible.

Problem S07.3. Consider the space of infinite sequences of real numbers


S = {(a0 , a1 , a2 , . . .) : an ∈ R, n = 0, 1, 2, . . .}.
For each pair if real numbers A and B, prove that the set of solutions (x0 , x1 , x2 , . . .) of the
linear recursion xn+2 = Axn+1 + Bxn , n = 0, 1, 2, . . . is a subspace of S of dimension 2.

Solution. Let S0 be set of all solutions to the recurrence relation. We first show that S0 is
indeed a subspace of S. Take x, y ∈ S0 . Put z = x + y. Then
zn+2 = xn+2 +yn+2 = Axn+1 +Bxn +Ayn+1 +Byn = A(xn+1 +yn+1 )+B(xn +yn ) = Azn+1 +Bzn .

17
Thus z ∈ S0 . Next for α ∈ R and x ∈ S0 , put w = αx. Then

wn+2 = αxn+2 = α(Axn+1 + Bxn ) = A(αxn+1 ) + B(αxn ) = Awn+1 + Bwn .

Thus w ∈ S0 . Hence S0 is closed under addition and multiplication by scalars and is thus a
subspace of S.
Next we show that if u, v ∈ S0 are such that u0 = 1, u1 = 0, v0 = 0, v1 = 1 then u, v form
a basis for S0 . It is clear that they are linearly independent and for x = (x0 , x1 , x2 , . . .) ∈ S0 ,
we have x = x0 u + x1 v. Indeed, x0 u + x1 v agrees with x in the first and second entries.
Assume that it agrees with x in the (n + 1)th and (n + 2)th entries for some n ≥ 0. Then

xn+1 = x0 un+1 + x1 vn+1 and xn+2 = x0 un+2 + x1 vn+2 .

Consider

xn+3 = Axn+2 + Bxn+1


= A(x0 un+2 + x1 vn+2 ) + B(x0 un+1 + x1 vn+1 )
= x0 (Aun+2 + Bun+1 ) + x1 (Avn+2 + Bvn+1 )
= x0 un+3 + x1 vn+3 .

Hence by induction it is true that x = x0 u + x1 v. Thus u, v span S0 . Hence dim(S0 ) = 2.

Problem S07.4. Suppose that A is a symmetric n × n real matrix and let λ1 , . . . , λ` be


the distinct eigenvalues of A. Find the sets
n 1/k o
X = x ∈ Rn : lim xt A2k x exists
k→∞

and n 1/k o
L= lim xt A2k x :x∈X .
k→∞

Solution. We prove that X = R and L = {0, λ21 , . . . , λ2` }.


n

By taking the zero vector for x, it is clear that the zero vector is in X and zero is in L.
We’ll focus on non-zero vectors henceforth.
Since A is symmetric, by the spectral theorem there is an orthonormal basis {x1 , . . . , xn }
for Rn consisting of eigenvectors of A. Suppose that µ1 , . . . , µn ∈ R are the corresponding
eigenvalues [the eigenvalues are real since A is symmetric; also repititions are allowed so this
is a slightly different list from λ1 , . . . , λ` ]. Let 0 6= x ∈ Rn . Then there are α1 , . . . , αn ∈ R
(not all zero) such that
x = α 1 x1 + . . . + α n xn .
Then
A2k x = α1 A2k x1 + · · · + αn A2k xn = α1 µ2k 2k
1 x1 + · · · + α n µ n xn

so by orthogonality,
xt A2k x = α12 µ2k 2k
1 + · · · + α n µn .

From here, it is clear that


xt A2k x ≥ αj2 µ2k
j

18
for all j = 1, . . . , n.
Let m ∈ {1, . . . , n} be an index satisfying αm 6= 0, and if αj 6= 0 for some j = 1, . . . , n,
then |µj | ≤ |µm |. That is, m is the index of the largest of the µ’s which has a non-zero
coefficient in the representation of x in this basis (note: such m always exists; it is not
necessarily unique, but that won’t matter). Then

xt A2k x = α12 µ2k 2k 2 2k


1 + · · · + αn µn ≤ nαm µm .

Then using our two bounds, we see that


2/k 2
1/k
αm µm ≤ xt A2k x 2 1/k 2
≤ (nαm ) µm .
1/k
Thus by the squeeze theorem, lim xt A2k x = µ2m .
k→∞
This shows that X = Rn . It also shows that for every non-zero x ∈ Rn , we have
1/k
lim xt A2k x = µ2j for some j = 1, . . . , n. Thus the eigenvalues of A are the only possible
k→∞
values for the limit. To see that each eigenvalue is indeed achieved, notice that
1/k
xtj A2k xj = µ2j
1/k
for all k ∈ N. Thus lim xtj A2k xj = µ2j for each j = 1, . . . , n.
k→∞

Problem S07.5. Let T be a normal linear operator on a finite dimensional complex inner
product space V . Prove that if v is an eigenvector of T , then v is also an eigenvector of the
adjoint T ∗ .

Solution. First, consider if S is any normal operator and x ∈ Cn , then

(Sx, Sx) = (x, S ∗ Sx) = (x, SS ∗ x) = (S ∗ x, S ∗ x).

Thus ||Sx|| = ||S ∗ x|| for all x ∈ Cn .


Let v be an eigenvector of T with corresponding eigenvalue λ ∈ C. Since T is normal, so
is (T − λI). Then

0 = ||(T − λI)v|| = ||(T − λI)∗ x|| = (T ∗ − λI)v .


Hence T ∗ v = λv so v is an eigenvector of T ∗ corresponding to eigenvalue λ.

Problem F07.3. Let V be a vector space and T a linear transformation such that T v and
v are linearly dependent for every v ∈ V . Prove that T is a scalar multiple of the identity
transformation.

Solution. If dim(V ) = 1, the result is trivial. Assume that dim(V ) > 1 Suppose x, y ∈ V
are linearly independent. Then there are scalars α, β, γ such that

T x = αx, T y = βy, T (x + y) = γ(x + y).

19
Then
0 = (γ − α)x + (γ − β)y.
But x, y were taken to be linearly independent, so α = γ = β. Since the same argument
would work for any linearly independent vectors, we see that T v = αv for all v ∈ V . Thus
T = αI.

Problem F07.12.
(a) Suppose that x0 < x1 < . . . < xn are points in [a, b]. Define linear functionals on Pn
(the space of all polynomials of degree less than or equal to n) by

`j (p) = p(xj ), j = 0, 1, . . . , n, p ∈ Pn .

Show that the set {`j }nj=0 is linearly independent.

(b) Show that there are unique coefficients cj ∈ R such that


Z b n
X
p(t)dt = cj `j (p)
a j=0

for all p ∈ Pn .
Solution.
(a) Let α0 , α1 , . . . , αn ∈ R be such that

α0 `0 + α1 `1 + · · · + αn `n = 0.

Put n
Y
pi (x) = (x − xm ), i = 0, 1, . . . , n.
m=0,m6=i

Then each pi is a polynomial of degree n and `i (pi ) 6= 0 since xi 6= xj when i 6= j.


However, `j (pi ) = 0 when i 6= j since (x − xj ) is a factor of pi when i 6= j. Thus for
each i = 0, 1, . . . , n,

α0 `0 (pi ) + α1 `1 (pi ) + . . . + αn `n (pi ) = 0 =⇒ αi `i (pi ) = 0 =⇒ αi = 0.

Hence `0 , `1 , . . . , `n are linearly independent.

(b) For any finite dimensional vector space V , we know dim(V ) = dim(V ∗ ). Here Pn has
dimension n + 1 and we have found n + 1 linearly independent members of (Pn )∗ .
Rb
Thus `0 , `1 , . . . , `n form a basis for (Pn )∗ . Since `(p) = a p(x)dx, p ∈ Pn defines a
linear functional on Pn , we know there are unique c0 , c1 , . . . , cn ∈ R such that ` =
c0 `0 + c1 `1 + · · · + cn `n . Hence
Z b Xn
p(x)dx = cj `j (p), for all p ∈ Pn .
a j=0

20
Problem S08.8. Assume that V is an n-dimensional vector space and that T is a linear
transformation T : V → V such that T 2 = T . Prove that every v ∈ V can be written
uniquely as v = v1 + v2 such that T (v1 ) = v1 and T (v2 ) = 0.

Solution. Note that if v ∈ im(T ), then v = T (w) for some w ∈ V and so

T (v) = T 2 (w) = T (w) = v.

Thus T fixes members of its image so T (v − T (v)) = 0 for all v ∈ V .


For any v ∈ V , write v = T (v) + (v − T (v)) ..= v1 + v2 . It’s clear that this is one way to
represent v in the desired way. Assume that v = v1 +v2 = x1 +x2 , where T (v1 ) = v1 , T (x1 ) =
x1 and T (v2 ) = 0, T (x2 ) = 0. Then

T (v) = T (v1 ) + T (v2 ) = T (x1 ) + T (x2 ) =⇒ T (v1 ) = T (x1 ) =⇒ v1 = x1 .

Then v1 + v2 = v1 + x2 =⇒ v2 = x2 . This gives uniqueness of such a representation.

Problem S08.9. Let V be a finite-dimensional vector space over R.

(a) Show that if V has odd dimension and T : V → V is a real linear transformation, then
T has a non-zero eigenvector v ∈ V .

(a) Show that for every even positive integer n, there is a vector space V over R of di-
mension n and a real linear transformation T : V → V such that there is no non-zero
v ∈ V that satisfies T (v) = λv for some λ ∈ R.

Solution.

(a) The characteristic polynomial pT of T is a polynomial over R of degree dim(V ) which


is odd. Every odd ordered polynomial over R has a root in R. We know this root (say
λ ∈ R) is an eigenvalue of T and thus there is a non-zero eigenvector v ∈ V such that
T (v) = λv. Hence T has an eigenvector.

(b) Let V = Rn for positive even n and let T be the left multiplication operator for
 
0 0 0 · · · 0 −1
1 0
 0 ··· 0 0  
0 1 0 ··· 0 0 
A = 0 0 .
 
 1 ··· 0 0  
 .. ... .. 
. . 
0 0 0 ··· 1 0

It is easy to see that T has characteristic polynomial pT (t) = (−t)n + (−1)n . But n is
even, so pT (t) = tn + 1. This equation has no roots in R when n is even and so there is
no real eigenvalue and thus no non-zero v ∈ Rn such that T (v) = λv for some λ ∈ R.

21
Problem F08.7. Suppose that T is a complex n×n matrix and that λ1 , . . . , λk are distinct
eigenvalues of T with corresponding eigenvectors v1 , . . . , vk . Show that v1 , . . . , vk are linearly
independent.

Solution. We use induction.


First, it is clear that {v1 } is a linearly independent set because eigenvectors are necessarily
non-zero.
Now assume that {v1 , . . . , v` } are linearly independent for some 2 ≤ ` < k. Let α1 , . . . , α` , α`+1 ∈
C be such that
α1 v1 + · · · + α` v` + α`+1 v`+1 = 0.
Then
(T − λ`+1 I)(α1 v1 + · · · + α` v` + α`+1 v`+1 ) = 0.
But (T − λ`+1 I)vi = (λi − λ`+1 )vi for i = 1, . . . , ` and (T − λ`+1 I)v`+1 = 0. Thus

α1 (λ1 − λ`+1 )v1 + · · · + α` (λ` − λ`+1 )v` = 0.

But these vectors are linearly independent by our inductive hypothesis, so α1 (λ1 − λ`+1 ) =
· · · = α` (λ` − λ`+1 ) = 0. However, since the eigenvalues are distinct, this implies that
α1 = · · · = α` = 0. Hence

0 = α1 v1 + · · · + α` v` + α`+1 v`+1 = α`+1 v`+1 =⇒ α`+1 = 0.

Thus {v1 , . . . , v` , v`+1 } is a linearly independent set.


By induction, we conclude that {v1 , . . . , vk } are linearly independent.

Problem F08.8. Must the eigenvectors of a linear transformation T : Cn → Cn span Cn ?

Solution. No. Take any non-diagonalizable matrix A ∈ Mn (C) and let T (x) = Ax, x ∈ Cn .
For example, if  
1 1 0
A = 0 1 1 .
0 0 2
Then, up to multiplication by scalars, the eigenvectors of A (and thus T ) are
   
1 1
v1 = 0 and v2 = 1
0 1

which clearly do not span C3 .

Problem F08.9.

(a) Prove that any linear transformation T : C → C must have an eigenvector.

(b) Is (a) true for any linear transformation T : Rn → Rn ?

22
Solution.
(a) By the fundamental theorem of algebra, pT (t) = det(T − tI) has a root λ ∈ C. Then
det(T − λI) = 0 so (T − λI) is a singular transformation. Hence there is a non-zero
v ∈ Cn such that (T − λI)(v) = 0 or T (v) = λv; that is, v is an eigenvector of T .
(b) No. Let  
0 −1
A=
1 0
and define T : R2 → R2 by T (x) = Ax, x ∈ R2 . Suppose T has an eigenvector
0 6= v ∈ R2 . Then there is λ ∈ R such that T (v) = λv. If v = (v1 , v2 ), this implies that
−v2 = λv1 and v1 = λv2 where at least one of v1 , v2 is non-zero. Asusme v1 6= 0. Then
by the second equation, neither λ nor v2 are zero. Also, by the first equation
−v2 = λv1 = λ2 v2 =⇒ λ2 = −1,
a contradiction to the fact that λ ∈ R. Hence there is no eigenvector for T .
Problem F08.11. Consider the Poisson equation with periodic boundary condition
∂ 2u
= f, x ∈ (0, 1),
∂x2
u(0) = u(1).
A second order accurate approximation to the problem is given by Au = ∆x2 f where
 
−2 1 0 ··· 0 1
 1 −2 1 0 ··· 0 
 
0 1 −2 1 0 · · ·
A= ,
 
.. .. ..
 . . . 
 
 0 ··· 0 1 −2 1 
1 0 ··· 0 1 −2

u = [u0 , u1 , . . . , un−1 ]t , f = [f0 , f1 , . . . , fn−1 ]t and ui ≈ u(xi ) with xi = i∆x, ∆x = 1/n and
fi = f (xi ) for i = 0, 1, . . . , n − 1.
(a) Show that A is singular.
(b) What conditions must f satisfy so that a solution exists?
Solution.
(a) Notice Av = 0 when v = [1 1 · · · 1]t , so A has a nontrivial null space and is thus
singular (in fact, up to scalar multiplication, this is the only vector in the null space
of A).
(b) We need f in the range of A for a solution to exist. The Fredholm alternative tells us
that range(A) = null(At )⊥ . But At = A, so we simply need f ∈ null(A)⊥ . By (a), this
is equivalent to
(v, f ) = 0 ⇐⇒ f0 + f1 + · · · + fn−1 = 0.

23
Problem F08.12. Consider the least square problem
min ||Ax − b||
x∈Rn

where A ∈ Rm×n , b ∈ Rm and m ≥ n. Prove that if x and x + αz (α 6= 0) are both minimiz-


ers, then z ∈ null(A).

Solution. Let b = b1 + b2 be the projection of b onto im(A); i.e., b1 is the unique vector in
im(A) such that b1 − b ∈ im(A)⊥ . We know that b1 is the closest vector to b which lies in
im(A). Then the minimizers of ||Ax − b|| are exactly those vectors x ∈ Rn such that Ax = b1 .
Let x be such a minimizer. Then for any y ∈ Rn , Ay ∈ im(A) and so (Ay, b1 − b) = 0. But
then
0 = (Ay, Ax − b) = (y, A∗ (Ax − b)).
Since y is arbitrary, this implies that A∗ (Ax − b) is orthogonal to all of Rn so A∗ (Ax − b) = 0
or A∗ Ax = A∗ b. That is, any minimizer x of ||Ax − b|| must satisfy the normal equations:
A∗ Ax = A∗ b.
Assume that both x and x + αz, α 6= 0 are minimizers of ||Ax − b|| . Then
A∗ b = A∗ Ax = A∗ A(x + αz) =⇒ αA∗ Az = 0 =⇒ A∗ Az = 0.
Taking the inner product of A∗ Az with z, we see
(z, A∗ Az) = 0 =⇒ (Az, Az) = 0 =⇒ ||Az||2 = 0 =⇒ Az = 0.
Thus z ∈ null(A).

Problem F09.4. Let V be a finite dimensional inner product space and let U be a sub-
space of V . Show that dim(U ) + dim(U ⊥ ) = dim(V ).

Suppose dim(U ) = n. Then dim(V ) = n + k for some k ∈ N0 . If k = 0, then U = V and


the result hold trivially. Assume k > 0. Let {x1 , . . . , xn } form an orthonormal basis for U .
We can extend this to an orthonormal basis {x1 , . . . , xn , y1 , . . . , yk } for V . If we can prove
that y1 , . . . , yk is an orthonormal basis for U ⊥ , then we will be done.
Since the set {x1 , . . . , xn , y1 , . . . , yk } is orthonormal, for any j = 1, . . . , k and i = 1, . . . , n,
we have (yj , xi ) = 0. Hince yj is orthogonal to each basis member for U and hence orthogonal
to U ; that is, yj ∈ U ⊥ for each j.
Take y ∈ U ⊥ . Then, y ∈ V so there are scalars α1 , . . . , αn , β1 , . . . , βk such that
y = α1 x1 + · · · + αn xn + β1 y1 + · · · + βk yk .
Since each xi ∈ U , we have (y, xi ) = 0. Alternately taking the inner product of y with each
xi , we find αi = 0 for each i. Thus
y = β1 y1 + · · · + βk yk .
Hence {y1 , . . . , yk } is a spanning set for U ⊥ ; it is also a linearly independent set since it is part
of the basis for V . Thus {y1 , . . . , yk } is a basis for U ⊥ so dim(U ⊥ ) = k so the claim is proven.

24
Problem F09.5. Show that if α1 , . . . , αn ∈ R are all different and some b1 , . . . , bn ∈ R
satisfy
n
X
bi eαi t for all t ∈ (−1, 1)
i=1

then necessarily b1 = · · · = bn = 0.

Solution. Let T : C ∞ (−1, 1) → C ∞ (−1, 1) be defined by

(T f )(t) = f 0 (t), t ∈ (−1, 1).

We see that for each αi , if we define fi ∈ C ∞ (−1, 1) by fi (t) = eαi t , t ∈ (−1, 1) then
T fi = αi fi . Hence the functions fi are eigenvectors of T corresponding to different eigenval-
ues. But eigenvectors of a linear operator corresponding to different eigenvalues are linearly
independent. Hence

b1 f 1 + · · · + bn f n = 0 =⇒ b1 = · · · = bn = 0

which is the desired conclusion.

Problem F09.12. Let n ≥ 2 and let V be an n-dimensional vector space over C with a
set of basis vectors e1 , . . . , en . Let T be the linear transformation of V satisfying

T (ei ) = ei+1 , i = 1, . . . , n − 1 and T (en ) = e1 .

(a) Show that T has 1 as an eigenvalue. Find an eigenvector with eigenvalue 1 and show
that it is unique up to scaling.

(b) Is T diagonalizable?

Solution.

(a) Let x = e1 + e2 + · · · + en ∈ V . Then T (x) = T (e1 ) + T (e2 ) + · · · + T (en ) = e2 +


e3 + · · · + e1 = e1 + e2 + · · · + en = x. Further x 6= 0 since the basis vectors are
linearly independent. Thus x is an eigenvector of T with eigenvalue 1. Conversely, if
v is an eigenvector of T corresponding to eigenvalue 1, then T (v) = v. Also there are
α1 , . . . , αn ∈ C such that v = α1 e1 + · · · + αn en . Then

α1 e1 + · · · + αn en = v = T (v) = α1 e2 + α2 e3 · · · + αn e1

which gives
(α1 − αn )e1 + (α2 − α1 )e2 + · · · (αn − αn−1 )en = 0.
But these vectors are linearly independent so

α1 − αn = 0, α2 − α1 = 0, · · · , αn − αn−1 = 0

which gives α1 = · · · = αn . Call this value α ∈ C. Then v = αe1 + · · · + αen = αx.


Hence the eigenvector is unique up to scaling.

25
(b) Since T (ej ) = ej+1 for all j = 1, . . . , n − 1 and T (en ) = e1 , the matrix [T ]B is given by

0 0 0 ···
 
0 1
1 0 0 · · · 
 
0 1 0 · · · 
[T ]B =  .. .. .
 
 . . 
 . .. .. 
 . 
0 0 0 ··· 1 0

Then pT (t) = det ([T ]B − tI) = ±(tn − 1). But tn − 1 has distinct roots for all n. Thus
the eigenvalues of T are distinct and so T is diagonalizable.

Problem S10.1. Let u1 , . . . , un be


Pan orthonormal basis for Rn and let y1 , . . . , yn be a
collection of vectors in R such that j=1 ||yj ||2 < 1. Show that u1 + y1 , . . . , un + yn form a
n n

basis for Rn .

Solution. Since invertible linear maps send one basis to another, it suffices to show that
there is an invertible linear map L : Rn → Rn such that L(uj ) = uj + yj for all j = 1, . . . , n.
Define a linear map, T : Rn → Rn by T (uj ) = −yj , j = 1, . . . , n (a linear map is uniquely
determined by how it treats basis elements).
Let x ∈ Rn . Then there are scalars α1 , . . . , αn such that x = nj=1 αj uj . Then
P


X n
||T (x)|| = αj T (uj )


j=1

X n
= αj y j


j=1
n
X
≤ |αj | ||yj ||
j=1
n
!1/2 n
!1/2
X X
≤ |αj |2 ||yj ||2 .
j=1 j=1

P 1/2
n 2
But by orthogonality of u1 , . . . , un , we have ||x|| = |αj |
j=1 . Thus we have that
Pn 2
||T || ≤ j=1 ||yj || < 1. But ||T || < 1 implies that I − T is invertible. Further, we see that
(I − T )(uj ) = uj + yj , j = 1, . . . , n. Hence u1 + y1 , . . . , un + yn form a basis for Rn .

Problem S10.3. Let S, T be two normal transformations in the complex finite dimensional
inner product space V such that ST = T S. Prove that there is a basis for V consisting of
vectors which are simultaneous eigenvectors of S and T .

Solution. Suppose that n = dim(V ). Let λ1 , . . . , λk be the distinct eigenvalues of S. Then


by the spectral theorem, the eigenvectors of S can be taken to form an orthonormal basis

26
for V which means that
Eλ1 ⊕ · · · ⊕ Eλk = V.
Consider, for v ∈ Eλi , we have Sv = λi v. Then

T Sv = T (λi v) =⇒ ST v = λi T v =⇒ S(T v) = λi (T v).



Thus T v ∈ Eλi . Hence Eλi is T -invariant. Then T E is a normal operator on Eλi so by the
λi
(i) (i)
spectral theorem, there is an orthonormal basis v1 , . . . , v`i for Eλi consisting of eigenvectors
of T . Then
[k [`i n o
(i)
vj
i=1 j=1

is a basis of V consisting of simultaneous eigenvectors of both S and T .

Problem S10.4.
(i) Let A be a real symmetric n × n matrix such that xt Ax ≤ 0 for every x ∈ Rn . Prove
that trace(A) = 0 implies A = 0.

(ii) Let T be a linear transformation in the complex finite dimensional vector space V with
an inner product. Suppose that T T ∗ = 4T − 3I where I is the identity transformation.
Prove that T is Hermitian positive definite and find all possible eigenvalues of T .
Solution. I’m have no idea why these two questions are grouped into one problem. As far
as I can see, they have nothing to do with each other and have elementary solutions which
are completely independent of the other.
(i) Let ei be the standard basis vectors. Then

eti Aei ≤ 0 =⇒ aii ≤ 0.

Then if the sum of the diagonal entries is zero, each entry must be zero; i.e., aii =
0, i = 1, . . . n. Next,

(ei + ej )t A(ei + ej ) ≤ 0 =⇒ aii + aij + aji + ajj ≤ 0 =⇒ aij + aji ≤ 0.

But since A is symmetric, this implies aij ≤ 0. Also

(ei − ej )t A(ei − ej ) = aii − aij − aji + ajj ≤ 0

which gives aij ≥ 0. Thus aij = 0. Since i, j were arbitrary, this gives A = 0.

(ii) We see (T T ∗ )∗ = (T ∗ )∗ T ∗ = T T ∗ . Thus

(4T − 3I)∗ = 4T − 3I =⇒ 4T ∗ − 3I = 4T − 3I =⇒ T∗ = T

so T is Hermitian. Then

T = 41 T T ∗ + 43 I = 14 T 2 + 34 I.

27
Then for any x ∈ V ,

(T x, x) = 14 (T 2 x, x) + 34 (x, x) = 14 (T x, T x) + 34 (x, x) ≥ 0

since inner products are positive definite. Further, if (T x, x) = 0 then (T x, T x) = 0


and (x, x) = 0. The latter can only happen if x = 0. Thus T is positive definite.
The functional equation for T gives

T 2 − 4T + 3I = 0 =⇒ (T − I)(T − 3I) = 0.

Then by the Cayley-Hamilton Theorem, the characteristic polynomial of T must divide


(t − 1)(t − 3). Thus the only possible eigenvalues of T are 1 and 3.
 
4 −4
Problem S10.6. Let A = .
1 0
(i) Find a Jordan form J of A and a patrix P such that P −1 AP = J.

(ii) Compute A100 and J 100 .

(iii) Find a formula for an when a0 = a, a1 = b and an+1 = 4an − 4an−1 .


Solution.
(i) The characteristic polynomial of A is pA (x) = x(x − 4) + 4 = x2 − 4x + 4 = (x − 2)2 .
Thus the sole eigenvalue of A is 2. We see N = A − 2I is not the zero matrix. Thus
there is x 6= 0 so that N x 6= 0. We see by inspection that x = (1, 0)t gives N x = (2, 1)t .
Put  
2 1
P = .
1 0
Then    
−1 0 −1 0 1
P =− =
−1 2 1 −2
and
        
−1 0 1 4 −4 2 1 1 0 2 1 2 1 .
P AP = = = .= J
1 −2 1 0 1 0 2 −4 1 0 0 2

(ii) We see     
22 1 2 1 4 4
J = = ,
0 2 0 2 0 4
and     
3 4 4 2 1 8 12
J = = .
0 4 0 2 0 8
From these, it is reasonable to guess that
 n 
n 2 n2n−1
J = .
0 2n

28
Indeed, assuming this holds for J n , we see
 n    n+1 n   n+1 
n+1 2 n2n−1 2 1 2 2 + n2n 2 (n + 1)2n
J = n = = .
0 2 0 2 0 2n+1 0 2n+1

Thus the formular holds by induction. Then A = P JP −1 =⇒ An = P J n P −1 . So


  n  
n 2 1 2 n2n−1 0 1
A =
1 0 0 2n 1 −2
 n+1   
2 (n + 1)2n 0 1
=
2n n2n−1 1 −2
 n n+1

(n + 1)2 −n2
= .
n2n−1 −(n − 1)2n
Then
   
100 101 · 2100 −100 · 2101 100 2100 100 · 299
A = and J = .
100 · 299 −99 · 2100 0 2100

(iii) The sequence an satisfies    


b
n an+1
A =
a an
so an = n2n−1 b − (n − 1)2n a.
Problem F10.5. Prove or disprove the following two statements. For any two subsets
U, W of a vector space V ,
(a) Span (U ) ∩ Span (W ) = Span (U ∩ W )
(b) Span (U ) + Span (W ) = Span (U ∪ W )
Solution.
(a) The statement is false. The sets
           
1 0 0 1 1 0
U= , , , and W = , ,
0 1 0 1 −1 0
provide a counterexample since
   
2 2 0 2 0
Span (U ) ∩ Span (W ) = R ∩ R = R 6= = Span = Span (U ∩ W ) .
0 0

(b) The statement is true. Take x + y ∈ Span (U ) + Span (W ) then x ∈ Span (U ) =⇒


x ∈ Span (U ∪ W ) and likewise for y. Hence, since Span (U ∪ W ) is a vector space, we
have x + y ∈ Span (U ∪ W ) and so Span (U ) + Span (W ) ⊂ Span (U ∪ W ).
Conversely, Span (U ) + Span (W ) clearly contains U ∪ W , but Span (U ∪ W ) is the
smallest vector space containing U ∪ W , so it must be the case that Span (U ∪ W ) ⊂
Span (U ) + Span (W ).

29
Problem F10.6. Let T be an invertible linear map on a finite dimensional vector space
V over a field F. Prove there is a polynomial f ∈ F[x] such that T −1 = f (T ).

Solution. By the Cayley-Hamilton Theorem, the linear operator satisfies pT (T ) = 0 where


pT is the characteristic polynomial. Put
pT (x) = α0 + α1 x + · · · + αn xn .
Specifically, since T is invertible, x = 0 is not a root of pT (x) and thus α0 6= 0. Then
pT (T ) = α0 I + α1 T + · · · + αn T n
where I is the identity map on V . Then
α1 αn n−1
−α0 I = α1 T + · · · + αn T n =⇒ T −1 = − I − ··· − T .
α0 α0
Hence T −1 is a polynomial expression of T .

Problem F10.7. Let V, W be inner product spaces over C such that dim(V ) ≤ dim(W ) <
∞. Prove that there is a linear transformation T : V → W satisfying (T (x), T (y))W = (x, y)V
for all x, y ∈ V .

Solution. . Suppose n ∈ N and k ∈ N ∪ {0} are such that dim(V ) = n, dim(W ) =


n + k. Let {v1 , . . . , vn } be an orthonormal basis for V and {w1 , . . . , wn , wn+1 , . . . , wn+k } be
an orthonormal basis for W . Define T : V → W such that T (vj ) = wj for j = 1, . . . , n and
T is linear (a linear map is completely determined by how it treats basis elements). For any
x, y ∈ V , there are α1 , . . . , αn , β1 , . . . , βn ∈ F such that
x = α1 v1 + · · · + αn vn , y = β1 v1 + · · · + βn vn .
Then
(T (x), T (y))W = (α1 w1 + · · · + αn wn , β1 w1 + · · · + βn wn )W
Xn X n X n
= αi β j (wi , wj )W = αi β i
i=1 j=1 i=1

by orthogonality. Also,
(x, y)V = (α1 v1 + · · · + αn vn , β1 v1 + · · · + βn vn )V
Xn X n X n
= αi β j (vi , vj )V = αi β i .
i=1 j=1 i=1

Thus (T (x), T (y))W = (x, y)V for all x, y ∈ V .

Problem F10.9. Consider the following iterative method:


xk+1 = A−1 (Bxk + c)
where      
2 0 2 1 1
A= , B= , c= .
0 2 1 2 1

30
(a) Assume the iteration converges. To what vector x does the iteration converge?

(b) Does the iteration converge for arbitrary initial vectors?


Solution.
(a) Assuming the iteration converges to x = (y, z)t , we must have

y = y + 21 z + 12 ,
x = A−1 (Bx + c) =⇒
z = 12 y + z + 21 .
 
−1
Thus x = .
−1

(b) Putting x0 = (a0 , a0 )t for some a0 ∈ R, we see that xn = (an , an )t , n ∈ N where

an = 12 (3an−1 + 1) = 32 an−1 + 12 , n ∈ N.

Then
2an = 3an−1 + 1 =⇒ 2an + 2 = 3an−1 + 3, n ∈ N.
Putting bn = an + 1, n ∈ N ∪ {0}, we get that

2bn = 3bn−1 , n ∈ N

from which an easy induction yields,


3 n

bn = 2
b0 , n ∈ N.

Then
3 n

an = 2
(a0 + 1) − 1, n ∈ N.
From here it is clear that the solution blows up as n → ∞ unless a0 = −1. Thus
the iteration does not converge for any initial vector of the form x0 = (a0 , a0 )t for
a0 ∈ R \ {−1}.
[Note: a classmate informs me that the iteration diverges for any initial vector other
that x0 = (−1, −1)t though I couldn’t be bothered to prove this.]
Problem F10.8. Let U, W be subspaces of a finite dimensional inner product space V .
Prove that (U ∩ W )⊥ = U ⊥ + W ⊥ .

Solution. Instead we prove that U ∩ W = (U ⊥ + W ⊥ )⊥ . This is sufficient because then

(U ∩ W )⊥ = ((U ⊥ + W ⊥ )⊥ )⊥ = U ⊥ + W ⊥

since the spaces are all finite-dimensional.


Let x ∈ U ∩ V . Then x ∈ U and x ∈ V . If y ∈ U ⊥ + W ⊥ , then y = y1 + y2 with
y1 ∈ U ⊥ , y2 ∈ W ⊥ and we see

(x, y) = (x, y1 ) + (x, y2 ) = 0 + 0 = 0.

31
Hence x is orthogonal to all vectors in U ⊥ + W ⊥ and so x ∈ (U ⊥ + W ⊥ )⊥ . Thus U ∩ V ⊂
(U ⊥ + W ⊥ )⊥ .
Let x ∈ (U ⊥ + W ⊥ )⊥ . Take y ∈ U ⊥ . Then y ∈ U ⊥ + W ⊥ and so (x, y) = 0. Thus x
is orthogonal to every member or U ⊥ so x ∈ (U ⊥ )⊥ = U. Likewise x ∈ (W ⊥ )⊥ = W . Thus
x ∈ U ∩ W so (U ⊥ + W ⊥ )⊥ ⊂ U ∩ W
Hence U ∩ W = (U ⊥ + W ⊥ )⊥ and the result follows.

Problem S11.2. Show that a positive power of an invertible matrix is diagonalizable if


and only if the matrix itself is diagonalizable.

Solution. Suppose that A ∈ Mn (C) is invertible.


If we suppose that A is diagonalizable, then there is diagonal D and invertible P in
Mn (C) such that A = P DP −1 . But then Am = P Dm P −1 for all m ∈ N and Dm is still
diagonal so Am is diagonalizable for all m ∈ N (the assumption that A is invertible is not
necessary for this direction).
Now suppose there is m ∈ N such that Am is diagonalizable. Then the minimial polyno-
mial of Am has the form
mAm (x) = (x − λ1 ) · · · (x − λk )
where λ1 , . . . , λk ∈ C are distinct and λi 6= 0 for i = 1, . . . , k since A (and thus Am ) is
invertible. Let
p(x) = mAm (xm ) = (xm − λ1 ) · · · (xm − λk ).
Then p(A) = mAm (Am ) = 0. Hence, the minimal polynomial of A must divide p(x). Let x0
be a root of p(x). Then xm m
0 = λi for some i = 1, . . . , k. Suppose that x0 = λ1 (the proof is
identical in other cases, but the details become more tedious to write down). Then

p0 (x0 ) = mxm−1
0 (xm m
0 − λ2 ) · · · (x0 − λk )
+ mxm−1
0 (xm m m
0 − λ1 )(x0 − λ3 ) · · · (x0 − λk )+
..
.
+ mxm−1
0 (xm m
0 − λ1 ) · · · (x0 − λk−1 )

where there are k summands and the ith summand omits (xm 0 −λi ) [this is simply the product
rule]. However, all of the summands except the first summand go to zero since xm 0 − λ1 = 0.
Thus
p0 (x0 ) = mxm−1
0 (xm m
0 − λ2 ) · · · (x0 − λk ).
0
However, xm0 = λ1 6= λ2 , . . . , λk and x0 6= 0 since λ1 6= 0. Thus p (x0 ) 6= 0 and so p(x) has
only simple roots. Since mA (x) (the minimal polynomial of A) divides p(x) it must have
only simple roots as well. Hence

mA (x) = (x − µ1 ) · · · (x − µ` )

where µ1 , . . . , µ` are distinct. This implies that A is diagonalizable.

Problem S11.5. Let A be an n × n matrix with real entries and let b ∈ Rn . Prove that
there exists x ∈ Rn such that Ax = b if and only if b is in the orthocomplement of the kernel

32
of the transpose of A.

Solution. “There exists x ∈ Rn such that Ax = b” is an equivalent statement to b ∈ Col(A)


and “b is in the orthocomplement of the kernel of the transpose of A” is the same as b ∈

Null (At ) . So the questions is asking us to prove that
⊥
Col(A) = Null At .
We prove this a bit more generally. Let V be an n-dimensional vector space and let T : V →
V be a linear transform with adjoint T ∗ . We prove that
im(T ) = ker(T ∗ )⊥ .
This clearly subsumes the above equality by letting V = Rn and T (x) = Ax, x ∈ Rn .
Let u ∈ ker(T ∗ ) and v ∈ im(T ). Then T (x) = v for some x ∈ V and
(v, u) = (T (x), u) = (x, T ∗ (u)) = (x, 0) = 0.
Hence u is orthogonal to every member of im(T ) so u ∈ im(T )⊥ . Thus ker(T ∗ ) ⊂ im(T )⊥ .
Let u ∈ im(T )⊥ . Then u is orthogonal to every member of im(T ). But T T ∗ (u) ∈ im(T ),
so
0 = (u, T T ∗ (u)) = (T ∗ (u), T ∗ (u)) = ||T ∗ (u)|| =⇒ T ∗ (u) = 0.
Thus u ∈ ker(T ∗ ) and so im(T )⊥ ⊂ ker(T ∗ ).
We conclude that
im(T )⊥ = ker(T ∗ ).
Since the two subspaces are equal, their orthocomplements are equal and thus
(im(T )⊥ )⊥ = ker(T ∗ )⊥ .
But for any subspace U of a finite-dimensional inner product space, we have (U ⊥ )⊥ = U .
Thus
im(T ) = ker(T ∗ )⊥ ,
which completes the proof.
Note, for a finite dimensional vector space V and a linear operator T : V → V , we have
ker(T ) = im(T ∗ )⊥ ,
ker(T ∗ ) = im(T )⊥ ,
im(T ) = ker(T ∗ )⊥ ,
im(T ∗ ) = ker(T )⊥ .
If V is not finite dimensional, in general we can only say
ker(T ) = im(T ∗ )⊥ ,
ker(T ∗ ) = im(T )⊥ ,
im(T ) ⊂ ker(T ∗ )⊥ ,
im(T ∗ ) ⊂ ker(T )⊥ .

33
Problem S11.6. Let V, W be finite dimensional real inner product spaces and let
A : V → W be a linear transform. Fix w ∈ W . Show that the elements v ∈ V for
which the norm ||Av − w|| is minimal are exactly the solutions to the equation A∗ Av = A∗ w.

Solution. Let w = w1 + w2 be the orthogonal projection of w onto im(A). That is, w1 ∈


im(A) and w2 ∈ im(A)⊥ and we know that w1 is the vector in im(A) closest to w. That is,
the minimizers of ||Av − w|| are those v ∈ V such that Av = w1 . We must show that these
satisfy the normal equation.
Suppose that v ∈ V is such that Av = w1 . We know that w2 = w − w1 = w − Av is
orthoronal to the image of A. Thus for all x ∈ V ,
(Ax, Av − w) = 0 =⇒ (x, A∗ (Av − w)) = 0).
Thus A∗ (Av − w) is orthogonal to the whole space V and thus A∗ (Av − w) = 0 and so
A∗ Av = A∗ w.
Conversely, suppose A∗ Av = A∗ w. It suffices to show that Av = w1 because w1 is the
closest member of im(A) to w. Consider
(Av − w1 , Av − w1 ) = (Av − w1 , Av − (w − w2 ))
= (Av − w1 , (Av − w) + w2 )
= (Av, Av − w) + (Av, w2 ) − (w1 , Av − w) − (w1 , w2 )
= (v, A∗ Av − A∗ w) + (Av, w2 ) − (w1 , Av − w) − (w1 , w2 ).
Further, we know that w2 is orthogonal to the image of A. Hence, (Av, w2 ) = (w1 , w2 ) = 0.
Also A∗ Av − A∗ w = 0 so
(Av − w1 , Av − w1 ) = −(w1 , Av − w).
However, w1 is in im(A). Thus w1 = Ax, for some x ∈ V . Hence
(Av − w1 , Av − w1 ) = −(Ax, Av − w) = −(x, A∗ Av − A∗ w) = 0.
Thus ||Av − w1 || = 0 so Av = w1 and so v is a minimizer of ||Av − w|| .

Problem F11.8. Assume that a complex matrix A satisfies


ker(A − λI) = ker((A − λI)2 )
for all λ ∈ C. Prove from first principles (i.e., without using canonical forms) that A is
diagonalizable.

Solution. Let λ be an eigenvalue of A. Suppose λ has algebraic multiplicity more ≥ 2.


Then λ is a root of the minimal polynomial more than once. That is
mA (x) = (x − λ)2 q(x)
for some polynomial q. Then forany vector v,
mA (A)v = 0 =⇒ (A − λI)2 q(A)v = 0.

34
But ker(A − λI) = ker((A − λI)2 ) so

(A − λI)q(A)v = 0.

Since this holds for all vectors v, this shows that (A − λI)q(A) = 0. But this contradicts the
minimality of mA . Thus all eigenvalues of A have algebraic multiplicity 1 so A is diagonal-
izable.

Problem F11.9. Let V be a finite dimensional complex inner product space and let
L : V → V be a self-adjoint linear operator. Suppose µ ∈ C, ε > 0 are given and assume
there is a unit vector x ∈ V such that

||L(x) − µx|| ≤ ε.

Prove there is an eigenvalue λ of L such that |λ − µ| ≤ ε.

Solution. By the spectral theorem, there is a orthonormal basis {v1 , . . . , vn } of V consisting


of eigenvectors of L (here,
P n = dim(V )). Let λ1 , . . . , λn be the corresponding eigenvalues
respectively. Then x = i (x, ei )ei and
n
X n
X
L(x) = (x, ei )L(ei ) = (x, ei )λi ei .
i=1 i=1

This leads to
2
X n
||L(x) − µx||2 = (x, ei )(λi − µ)ei


i=1
n n
!
X X
= (x, ei )(λi − µ)ei , (x, ej )(λj − µ)ej
i=1 j=1
n X
X n
= (x, ei )(x, ej )(λi − µ)(λj − µ)(ei , ej )
i=1 j=1
n
X
= |(x, ei )|2 |λi − µ|2 , by orthonormality.
i=1

If for every i, we had


|λi − µ| > ε
then n
2
X
||L(x) − µx|| > ε 2
|(x, ei )|2 = ε2 ||x|| = ε2 .
i=1

However, this contradicts our assumption that ||L(x) − µx|| ≤ ε. Hence there is some
i = 1, . . . , n such that |λi − µ| ≤ ε.

35
Problem F11.10. Let A be a 3 × 3 real matrix with A3 = I. Show that A is similar to a
matrix of the form  
1 0 0
0 cos θ − sin θ .
0 sin θ cos θ
What values of θ are possible?

Solution. From A3 = I, we get det(A)3 = 1 so det(A) = 1. Also since the characteristic


polynomial of A has degree 3, it has a real root. Thus A has a real eigenvalue, λ. We see
that λ3 is an eigenvalue of A3 = I and thus λ3 = 1 and so λ = 1.

If all eigenvalues of A are real then they are all 1 by the reasoning above. Since A3 = I,
we know A is diagonalizable and thus A is similar to I. But this implies A = I. Then A is
of the above form with θ = 0 so the claim clearly holds.
Otherwise, since complex eigenvalues of a real matrix come in conjugate pairs, besides
1, A has eigenvalues of the form µ, µ where µ ∈ C. Then 1 = det(A) = λµµ = |µ|2 and so
µ = eiθ = cos θ + i sin θ for some θ ∈ [0, 2π). The eigenvectors corresponding to µ, µ will also
be a conjugate pair. Let the eigenvector of µ be w = w1 + iw2 . where w1 , w2 ∈ R3 . Then

Aw = µw =⇒ Aw1 = cos θw1 − sin θw2 and Aw2 = sin θw1 + cos θw2 .

Put P = [v w1 w2 ]. Then
 
1 0 0
AP = (Av | Aw1 | Aw2 ) = (v | cos θw1 − sin θw2 | sin θw1 + cos θw2 ) = P 0 cos θ − sin θ
0 sin θ cos θ

which shows that A is similar to a matrix of the desired form. There are no restrictions put
on θ throughout the derivations, so it could be any θ ∈ R. Of course, θ ∈ [0, 2π) will suffice.

Problem F11.11.

(a) State and prove the rank-nullity theorem.

(b) Suppose U, V, W are finite dimensional vector spaces over R and that T : U → V
and S : V → W are linear operators. Suppose that T is injective, S is surjective
and S ◦ T = 0. Prove that im(T ) ⊂ ker(S) and that dim(V ) − dim(U ) − dim(W ) =
dim(ker(S)/im(T )).

(a) Rank-Nullity Theorem. Let U, V be finite dimensional vector spaces and T : U → V


be a linear map. Then

dim(im(T )) + dim(ker(T )) = dim(U ).

Proof. Assume dim(ker(T )) = n and dim(U ) = n + k. Let {x1 , . . . , xn } be a basis for


ker(T ). We can extend this to a basis of U : {x1 , . . . , xn , u1 , . . . uk }. Take v ∈ im(T ).

36
Then v = T (u) for some u ∈ U . But we have a basis for U , so there are scalars
α1 , . . . , αn , β1 , . . . , βk such that

u = α1 x1 + · · · + αn xn + β1 u1 + · · · + βk uk .

Then

v = T (u) = α1 T (x1 ) + · · · + αn T (xn ) + β1 T (u1 ) + · · · + βk T (uk )


= β1 T (u1 ) + · · · + βk T (uk ) (since xi ∈ ker(T ), i = 1, . . . , n).

Thus {T (u1 ), . . . , T (uk )} is a spanning set for im(T ). Now assume that γ1 , . . . , γk are
scalars such that
γ1 T (u1 ) + · · · + γk T (uk ) = 0.
Then
T (γ1 u1 + · · · + γk uk ) = 0 =⇒ γ1 u1 + · · · + γk uk ∈ ker(T ).
Then there are scalars δ1 , . . . , δn ,

γ1 u1 + · · · + γk uk = δ1 x1 + · · · + δn xn =⇒ γ1 u1 + · · · + γk uk − δ1 x1 − · · · − δn xn = 0.

But {x1 , . . . , xn , u1 , . . . uk } form a basis and are thus linearly independent. Hence,
γ1 = · · · = γk = 0 so {T (u1 ), . . . , T (uk )} is a linearly independent set.
Thus {T (u1 ), . . . , T (uk )} is a basis for im(T ) so dim(im(T )) = k and the result is
proven.

(b) Take x ∈ im(T ). Then x = T (u) for some u ∈ U . But then S(x) = (S ◦ T )(u) = 0
since S ◦ T is the zero transformation. Thus x ∈ ker(S) so im(T ) ⊂ ker(S).
Apply the rank-nullity theorem to T and S to see

dim(im(T )) + dim(ker(T )) = dim(U ),


dim(im(S)) + dim(ker(S)) = dim(V ).

But dim(ker(T )) = 0 since T is injective and dim(im(S)) = dim(W ) because S is


surjective. Hence

dim(im(T )) = dim(U ),
dim(ker(S)) = dim(V ) − dim(W ).

Then

dim(ker(S)/im(T )) = dim(ker(S)) − dim(im(T )) = dim(V ) − dim(W ) − dim(U )

which is exactly the conclusion we needed to prove.

37
Problem S12.7. Let F be a finite field of p elements and V be an n-dimensional vector
space over F . Compute the number of invertible linear maps from V → V .

Solution. The matrix form of any invertible map must be invertible and thus must have
linearly independent columns. The choice for elements in the first column is arbitrary except
they can’t all be zero. Thus there are pn − 1 choices. The choice for elements in the next
column simply cannot be a scalar multiple of the first. Thus there are pn − p choices. The
next column cannot be a linear combination of the first two so there are pn − p · p = pn − p2
choices. Similarly, there are pn − pk−1 choices for the k th column. Multiplying these, we see
there are
n−1
Y
n n n n−1
(p − 1)(p − p) · · · (p − p ) = (pn − pk )
k=0

invertible linear maps on V .

Problem S12.9. Let a1 = 1, a2 = 4, an+2 = 4an+1 − 3an for all n ≥ 1. Find a 2 × 2 matrix
A such that    
n 1 an+1
A = .
0 an
Use the eigenvalues of A to determine the limit

lim (an )1/n .


n→∞

Solution. From n = 1, we see that A has the form


 
4 b
A= .
1 d

Then  
2 16 + b 4b + bd
A = .
4 + d b + d2
From n = 2,        
21 13 16 + b 13
A = =⇒ = ,
0 4 4+d 4
so  
4 −3
A= .
1 0
We see that det(A − λI) = λ(λ − 4) + 3 = (λ − 1)(λ − 3). Thus λ1 = 1 and λ2 = 3 are
eigenvalues of A with corresponding eigenvectors
   
1 3
v1 = , v2 = .
1 1

Notice that      
1 1 1 1 3 .
=− + .= x 1 + x 2 ,
0 2 1 2 1

38
where x1 , x2 are still eigenvectors corresponding to λ1 , λ2 respectively. Finally, for any n ≥ 1,
   
an+1 n 1
=A = An (x1 + x2 ) = λn1 x1 + λn2 x2 ,
an 0

which gives an = 21 3n − 21 . Thus


lim (an )1/n = 3.
n→∞

Problem S12.11.

(a) Find a polynomial P (x) of degree 2 such that P (A) = 0 for


 
1 3
A= .
4 2

(b) Prove that P (x) from part (a) is unique up to scalar multiplication.

Solution.

(a) The characteristic polynomial of A is P (x) = (x − 1)(x − 2) − 12 = x2 − 3x − 10. We


check
       
2 13 9 3 9 10 0 0 0
P (A) = A − 3A − 10I = − − = = 0.
12 16 12 6 0 10 0 0

(We knew to check the characteristic polynomial because the Cayley-Hamilton The-
orem tells us that the characteristic polynomial of a matrix always annihilates the
matrix.)

(b) It is clear that there is no first degree polynomial Q such that Q(A) = 0 because A is
not a scalar multiple of the identity.
Suppose Q(A) = 0. By the above, this implies that Q is a second degree polynomial.
Then there is α ∈ R such that αQ is a monic polynomial. Then (P − αQ)(A) = 0 and
P −αQ is a first degree polynomial or a constant polynomial. However, as stated above,
there is no first degree polynomial which annihilates A. Hence P − αQ is constant so
(P − αQ)(A) = 0 =⇒ P − αQ = 0. Thus P = αQ so the polynomial is unique up
to multiplication by a constant.

Problem F12.7. Let A be an invertible m × m matrix over C and suppose the set of
powers An of A is bounded for n ∈ Z. Prove that A is diagonalizable.

Solution. Let λ ∈ C be an eigenvalue of A. Then λ 6= 0 since A is invertible. Further,


there is a unit vector v ∈ C such that Av = λv. Then An v = λn v for all n ∈ Z. Then we see

||An || ≥ ||An v|| = ||λn v|| = |λ|n .

6 1, then |λ|n is unbounded for n ∈ Z and thus An would be unbounded for n ∈ Z; a


If |λ| =
contradiction. Thus all eigenvalues λ of A satisfy ||λ|| = 1.

39
Assume A is not diagonalizable. Then the Jordan canonical form of A has a block of at
least size 2. Then there is an eigenvalue λ of A and two vectors v, w such that Av = λv and
Aw = v + λw. This implies

A2 w = Av + λAw = λv + λ(v + λw) = 2λv + λ2 w.

Further
A3 w = 2λAv + λ2 Aw = 2λ2 v + λ2 (v + λw) = 3λ2 v + λ3 w.
Indeed, proceeding by induction, we have

An w = nλn−1 + λn w.

Then

||An || ≥ ||An w|| = nλn−1 v + λn w ≥ nλn−1 ||v|| − |λn | ||w|| = n ||v|| − ||w|| .

Since this holds for all n ∈ N, letting n → ∞ shows that the powers of A are not bounded,
a contradiction. Thus A is diagonalizable.

Problem F12.10. Let A be a linear operator on a four dimensional complex vector space
that satisfies the polynomial equation P (A) = A4 + 2A3 − 2A − I = 0, there I is the identity
operator on V . Suppose that |tr(A)| = 2 and that dim(range(A + I)) = 2. Give a Jordan
canonical form of A.

Solution. Let’s factor P (x). By inspection, we see that 1 is a root. So P (x) = (x − 1)(x3 +
αx2 + βx + 1). Expanding, we see α − 1 = 2 and β − α = 0 (from the x3 and x2 term
respectively). Then α = β = 3 so P (x) = (x − 1)(x + 1)3 . Thus possible eigenvalues of A are
1, −1. By the rank-nullity theorem, dim(ker(A + I)) = 4 − dim(range(A + I)) = 2. Thus
the algebraic multiplicity of −1 is at least 2. Then tr(A) = ±2 forces −1 to have algebraic
multiplicity 3. Thus A is not diagonalizable. A canonical form has 3 Jordan blocks; two of
these are 1 × 1 blocks containing 1 and −1. The third block is 2 × 2 with −1 as the two
diagonal elements and 1 on the superdiagonal. That is a canonical form J of A is
 
1 0 0 0
0 −1 0 0
J = 0 0 −1 1  .

0 0 0 −1

Problem F12.12. Let M be an n × m matrix. Prove that the row rank of M equals
the column rank of M . Interpret this result as an equality of the dimensions of two vector
spaces naturally attached to the map defined by M .

Solution. Let T : Rm → Rn be the associated linear map: T (x) = M x, x ∈ Rm . Also


define T ∗ : Rn → Rm by T ∗ (y) = M t y, y ∈ Rn .

40
Then im(T ) = col(M ) and im(T ∗ ) = col(M t ) = row(M ). Thus the problem boils down
to showing that dim(im(T )) = dim(im(T ∗ )).
Let y1 , . . . , yk ∈ Rn be a basis for im(T ). Then T ∗ (y1 ), . . . , T ∗ (yk ) ∈ im(T ∗ ). Take
α1 , . . . αk ∈ R such that
α1 T ∗ (y1 ) + · · · + αk T ∗ (yk ) = 0.
Then
T ∗ (α1 y1 + · · · + αk yk ) = 0
so α1 y1 + · · · + αk yk ∈ ker(T ∗ ). Recall that ker(T ∗ ) = im(T )⊥ , so α1 y1 + · · · + αk yk ∈ im(T )⊥ .
However since y1 , . . . , yk ∈ im(T ), we know that α1 y1 + · · · + αk yk ∈ im(T ). Hence, then

α1 y1 + · · · αk yk ∈ im(T ) ∩ im(T )⊥

so
α1 y1 + · · · αk yk = 0.
But y1 , . . . , yk form a basis, so α1 = · · · = αk = 0. Hence T ∗ (y1 ), . . . , T ∗ (yk ) are linearly
independent so dim(im(T ∗ )) ≥ k = dim(im(T )).
Using the same argument but starting with a basis of im(T ∗ ) shows that dim(im(T ∗ )) ≤
dim(im(T )), thus the values are equal and so the row rank of M is equal to its column rank.
The interpretation as an equality of dimensions of two vector spaces naturally attached
to M would be

dim(im(T )) = dim ker(T )⊥ or dim(col(M )) = dim null(M )⊥


 

since T is the left multiplication operator for M .

Problem S13.8. Let V, W be finite dimensional inner product space and T : V → W be


a linear map.

(a) Define the adjoint map T ∗ : W → V .

(b) Show that if the matrices are written relative to orthonormal bases of V and W then
the matrix of T ∗ is the transpose of the matris of T .

(c) Show that the kernel of T ∗ is the orthogonal complement of the range of T .

(d) Use (b) and (c) to prove that the row rank of a matrix is the same as the column rank
of the matrix.

Solution.

(a) The adjoint T ∗ : W → V is the unique linear map such that

(T v, w)W = (v, T ∗ w)V

for all v ∈ V , w ∈ W , where (·, ·)V , (·, ·)W are the inner products on V and W
respectively.

41
(b) Let {v1 , . . . , vn } be an orthonormal basis for V and {w1 , . . . , wm } be an orthonormal
basis for W . For each vj , there are scalars α1j , . . . , αmj such that
m
X
T (vj ) = aij wi .
i=1

Then the matrix of T is given by [T ] = (aij ) where i = 1, . . . , m, j = 1, . . . , n. Similarly,


for each wi there are scalars β1i , . . . , βni such that
n
X
T ∗ (wi ) = βji vj .
j=1

Then [T ∗ ] = (βji ). We need to show that βji = αij and this will imply that [T ] = [T ∗ ]t .
We see by orthogonality that, αij = (T vj , wi ). Then αij = (vj , T ∗ wi ) by definition of
the adjoint. But then by orthogonality, αij = βji . Hence [T ] = [T ∗ ]t .
(c) Take w ∈ ker(T ∗ ). Let z ∈ im(T ). Then z = T v for some v ∈ V . Then
(z, w) = (T v, w) = (v, T ∗ w) = (v, 0) = 0.
Thus w is orthogonal to z. Since z ∈ im(T ) was arbitrary, this shows that w ∈ im(T )⊥ .
Take w ∈ im(T )⊥ . Then w is orthogonal to every member of the image of T . In
particular T (T ∗ w) ∈ im(T ). Thus
0 = (T (T ∗ w), w) = (T ∗ w, T ∗ w) = ||T ∗ w||2 =⇒ T ∗ w = 0.
Thus w ∈ ker(T ∗ ).
We conclude that ker(T ∗ ) = im(T )⊥ .
(d) Translating (c) into matrix form using (b), we see that for A ∈ Mn,m (C), we have
null(At ) = col(A)⊥ .
We see that col(At ) = row(A). So we must show that col(A) = col(At ).
Let {x1 , . . . , xk } be a basis for the column space of A. Then At x1 , . . . , At xk are in the
column space of At . Let α1 , . . . , αk ∈ C be such that
α1 At x1 + · · · + αk At xk = 0.
Then
At (α1 x1 + · · · + αk xk ) = 0
so α1 x1 +· · ·+αk xk ∈ null(At ). But by (c), this implies that α1 x1 +· · ·+αk xk ∈ col(A)⊥ .
However α1 x1 + . . . + αk xk is also in col(A). Thus it is orthogonal to itself and so
it is zero. But, x1 , . . . , xk are linearly independent so α1 = · · · = αk = 0. Hence
At x1 + · · · + At xk are linearly independent in col(At ) = row(A). Thus dim(row(A)) ≥
k = dim(col(A)).
Making the same argument but beginning with a basis for the column space of At
shows that dim(row(A)) ≤ dim(col(A)). Thus dim(row(A)) = dim(col(A)) so the row
rank and column rank of A are the same.

42
Problem S14.1.
(a) Find a real matrix A whose minimal polynomial is equal to

t4 + 1.

(b) Show that the real linear map determined by A has no non-trivial invariant subspace.
Solution.
(a) Put  
0 1 0 0
0 0 1 0
A=
0
.
0 0 1
−1 0 0 0
Then
     
0 0 1 0 0 0 0 1 −1 0 0 0
 0 0 0 1  −1 0 0 0  0 −1 0 0
A2 = 
−1 0
, A3 = 
 0 −1 0
, A4 =  .
0 0 0 0 0 −1 0 
0 −1 0 0 0 0 −1 0 0 0 0 −1

Thus A4 + I = 0. Further, A3 , A2 , A, I are linearly independent, so there is no third


degree polynomial which annihilates A. Thus t4 + 1 is the minimal polynomial of A.

(b) This isn’t true. For example, if


 
0 1 0 0 0 0 0 0
0 0 1 0 0 0 0 0
 
0 0 0 1 0 0 0 0
 
−1 0 0 0 0 0 0 0
A=
0

 0 0 0 0 1 0 0

0 0 0 0 0 0 1 0
 
0 0 0 0 0 0 0 1
0 0 0 0 −1 0 0 0

then A still has minimal polynomial t4 + 1 but Span (e1 , e2 , e3 , e4 ) is invariant under
A. Even if A has to be 4 × 4, we can only prove that A has no 1 or 3 dimensional
invariant subspaces. There are cases where A has a 2 dimensional invariant subspace.
Problem S14.2. Suppose that S, T : V → V are linear where V is a finite dimensional
vector space over R. Show that

dim(im(S)) + dim(im(T )) ≤ dim(im(S ◦ T )) + dim(V ).

Solution. Adding dim(ker(S)) and dim(ker(T )) to both sides and using the rank nullity
theorem, we see that the given inequality is equivalent to

dim(V ) ≤ dim(im(S ◦ T )) + dim(ker(S)) + dim(ker(T )).

43
Next adding dim(ker(S ◦ T )) to both sides, we see the original inequality is equivalent to

dim(ker(S ◦ T )) ≤ dim(ker(S)) + dim(ker(T )).

If we can prove this last inequality, we will have proven the first.
Consider, if x ∈ ker(T ) then (S ◦ T )(x) = S(T (x)) = S(0) = 0 so x ∈ ker(S ◦ T ). Thus
ker(T ) is a subspace of ker(S ◦ T ). Let {x1 , . . . , xk } be a basis of ker(T ). Extend this to a
basis {x1 , . . . , xk , y1 , . . . , y` } of ker(S ◦ T ). Then

(S ◦ T )(yi ) = S(T (yi )) = 0

for each i, so T (yi ) ∈ ker(S). Suppose α1 , . . . , α` ∈ R are such that

α1 T (y1 ) + · · · + α` T (y` ) = 0.

Then
T (α1 y1 + · · · α` y` ) = 0
so α1 y1 + · · · α` y` ∈ ker(T ). Then there are β1 , . . . , βk ∈ R such that

α1 y1 + · · · + α` y` = β1 x1 + · · · + βk xk =⇒ α1 y1 + · · · + α` y` − β1 x1 − · · · − βk xk = 0.

But these vectors form a basis for ker(S ◦ T ) so they are linearly independent. Hence all
coefficients are zero. Thus {T (y1 ), . . . , T (y` )} is a lineary independent set in ker(S). Hence
dim(ker(S)) ≥ `. Then

dim(ker(S ◦ T )) = ` + k = ` + dim(ker(T )) ≤ dim(ker(S)) + dim(ker(T )).

The result follows.

Problem S14.3. Suppose that A, B ∈ Mn (C) satisfy AB − BA = A. Show that A is not


invertible.

Solution. Suppose that A is invertible. Let λ1 , . . . , λ` ∈ C be the distinct eigenvalues of B


ordered such that
Re(λ1 ) ≤ Re(λ2 ) ≤ · · · ≤ Re(λ` ).
Multiplying by A−1 on the right, we see

ABA−1 − B = I =⇒ ABA−1 = B + I.

Thus B is similar to B+I. However, B+I has λ` +1 as an eigenvalue and Re(λ` +1) > Re(λj )
for all j = 1, . . . , `. Thus B and B + I do not have the same eigenvalues and thus are not
similar; a contradiction. Hence A is not invertible.

Problem S14.4. Suppose A, B ∈ Mn (C). Show that the characteristic polynomials of AB


and BA are equal.

44
Solution. Suppose B is invertible. Then
AB = (B −1 B)AB = B −1 (BA)B.
Thus AB and BA are similar so they have the same characteristic polynomial.
If B is not invertible, let λ be the non-zero eigenvalue of B with least real part. Then for
0 < t < |Re(λ)| , Bt = B + tI is invertible [note: if all eigenvalues of B have zero real part,
then this holds for all t > 0]. Then ABt has the same characteristic polynomial as Bt A.
However, the characteristic polynomials of a matrix is a continuous function of the matrix
itself [this is because the determinant map is smooth]. Thus taking the limit as t → 0, we
see that AB and BA have the same characteristic polynomial.

Problem S14.6. Show that if A ∈ Mn (C) is normal then A∗ = P (A) for some P ∈ C[x].

Solution. Since A is normal, by the spectral theorem, we can unitarily diagonalize it:
A = U DU ∗ where U is unitary, D is diagonal. Then for any polynomial Q, we have Q(A) =
U Q(D)U ∗ . Thus we reduce the problem to finding P ∈ C[x] such that
U DU ∗ = U P (D)U ∗ ⇐⇒ D = P (D).
However, a polynomial acting on a diagonal matrix acts individually on each diagonal el-
ement. Let λ1 , . . . , λ` ∈ C be the distinct eigenvalues of A. Then these are the diagonal
elements of D and the λ1 , . . . , λ` are the diagonal elements of D. Thus all we need a polyno-
mial which satisfies P (λj ) = λj for all 1, . . . , `. Such a polynomial certainly exists and can
be constructed using Lagrange interpolants. Hence A∗ can be expressed as a polynomial in A.

Problem F14.7. Among all solutions to the system


   
1 1 1 1 2
 2 3 5 7 x =  7 
−2 1 1 3 −1
find the solution with minimal length.

Solution. We notice    
1 1 1 1 1 1 1 1
 2 3 5 7 ∼ 0 1 3 5 .
−2 1 1 3 0 0 0 0
Thus all solutions of the given system are of the form
     
−1 2 4
3 −3 −5
x=  0  + s 1  + t 0 
    

0 0 1
for some s, t ∈ R (in fact we could replace (−1, 3, 0, 0)t with any particular solution). Thus
we need to minimize
f (s, t) = (−1 + 2s + 4t)2 + (3 − 3s − 5t)2 + s2 + t2

45
among all (s, t) ∈ R2 . We know the minima must annihilate the first derivatives, so

4(−1 + 2s + 4t) − 6(3 − 3s − 5t) + 2s = 0 and 8(−1 + 2s + 4t) − 10(3 − 3s − 5t) + 2t = 0.

Simplifying, this gives


23
     11 
1 14
s
42 = 14
19 .
1 23
t 23

After copious amounts of infuriating and mind-numbing algebra, this yields


25 13
s= , t=
59 59
so the solution of minimal length is
 
43
1 37

x= .
59 25
13

Problem F14.8. Compute the eigenvalues of the n × n matrix


 
k 1 1 ··· 1
1 k
 1 · · · 1
M = 1 1
 k · · · 1.
 .. .. .. . . .. 
. . . . .
1 1 1 ··· k

Use the eigenvalues to compute det(M ).

Solution. We notice that λ = k − 1 is an eigenvalue and M − λI has rank 1. Thus the


algebraic multiplicity of λ is n − 1. Hence we only need one more eigenvalue. By inspection
if x = (1, 1, . . . , 1)t then M x = (k + (n − 1))x. Thus k + (n − 1) = (k − 1) + n is another
eigenvalue.
The determinant is the product of the eigenvalues so det(M ) = (k − 1)n + n(k − 1)n−1 .
[Note: a less “inspective” approach might use induction, but I couldn’t figure out how to
do that.]

Problem F14.9. Suppose A 6= 0 is an n × n complex matrix. Prove that there is a matrix


B such that B and A + B have no eigenvalues in common.

Solution. Not sure.


Problem F14.10. What is the largest possible number of 1’s an invertible n × n matrix
with entries in {0, 1} can have? You must show this number is possible and that no larger
number is possible.

Solution. The answer is n2 − n + 1.

46
First we show that no larger number is possible. Suppose an n × n matrix with entries
in {0, 1} has at least n2 − n + 2 entries that are 1. Then there are less than n − 2 zeros in
the matrix. But this means that at least two columns do not have a zero. These columns
will be the linearly dependent and thus the matrix is not invertible.
Consider the matrix  
1 1 1 ··· 1
0 1 1 · · · 1
 
1 0 1 · · · 1
A= .
 .. .. . . . . .. 
. . . . .
1 1 ··· 0 1
That is, A is a matrix full of 1’s but with zeros on the first subdiagonal. Then there are
n2 − n + 1 entries that are 1. Consider solving Ax = 0. By subtracting the second row from
the first we would find x1 = 0. Then by subtracting the third row from the second, we would
find x2 = 0. Likewise we would find xk = 0, k = 1, . . . , n. Thus Ax = 0 has only the trivial
solution so A is invertible. Thus we can find an invertible matrix with n2 − n + 1 entries
which are 1.

Problem F14.11. Suppose a 4 × 4 integer matrix has four distinct real eigenvalues
λ1 > λ2 > λ3 > λ4 . Prove that λ21 + λ22 + λ23 + λ24 ∈ Z.

Solution. Recall that the trace of a matrix is the sum of the eigenvalues. Since A has
integer entries so does A2 . Thus the trace of A2 is an integer since the diagonal elements of
A2 are all integers. However, the eigenvalues of A2 are λ21 , λ22 , λ23 , λ24 so their sum must equal
the trace of A2 and hence λ21 + λ22 + λ23 + λ24 ∈ Z.
[Apparently the assumptions that the eigenvalues are real and distinct aren’t necessary.]

1
Problem F14.12. Prove that the matrix A = (aij ) given by aij = i+j−1
, i, j = 1, . . . , n
is positive definite.

Solution. The matrix is the Gram matrix for the basis {1, x, . . . , xn−1 } of Pn−1 [0, 1] with
the inner product Z 1
(p, q) = p(x)q(x)dx, p, ∈ Pn−1 [0, 1].
0
All Gram matrices are positive definite. To see this, let {v1 , . . . , vn } be a linearly independent
set in an inner product space and let A be the matrix given by
Aij = (vi , vj ).
Then for any x ∈ Rn , x = (x1 , . . . , xn )t , we see
n n n n
!
X X X X
xt Ax = xi xj (vi , vj ) = (xi vi , xj vj ) = xi v i , xj v j = (v, v) ≥ 0
i,j=1 i,j=1 i=1 j=1

where v = ni=1 xi vi . Further, there is equality iff v = 0 which happens iff x = 0 since
P
{v1 , . . . , vn } is a linearly independent set. Thus A is positive definite.

47
Problem S15.7. Let

f (x, y, z) = 9x2 + 6y 2 + 6z 2 + 12xy − 10xz − 2yz.

Does there exist a point (x, y, z) such that f (x, y, z) < 0?

Solution. I assume they mean (x, y, z) ∈ R3 . In this case the answer is no. We see that
any critical point (x, y, z) of f must satisfy

∂f
0= (x, y, z) = 18x + 12y − 10z,
∂x
∂f
0= (x, y, z) = 12x + 12y − 2z,
∂y
∂f
0= (x, y, z) = −10x − 2y + 12z.
∂z
However, the matrix  
9 6 −5
A= 6 6 −1
−5 −1 6
is invertible since

det(A) = 9(37) − 6(41) − 5(24) = 333 − 246 − 120 = 333 − 366 = −33.

Thus the only solution to the above system is (0, 0, 0). Thus (0, 0, 0) is either a global
maximum or a global minimum. It is easy to see that f (1, 1, 1) > 0 whereas f (0, 0, 0) = 0 so
(0, 0, 0) must be a global minimum. Hence there is no point (x, y, z) such that f (x, y, z) < 0.
[I’m not sure what the indented solution was here but I’m fairly certain the correct ap-
proach was to factor the polynomial into a sum of squares. I couldn’t figure out how to do
this. The function f is the quadratic form which is induced by A as defined above so that
problem is actually to prove that this matrix is positive definite.]

Problem S15.8. Prove or disprove the following claims:

(a) Matrices with determinant 1 are dense in the set of all 3 × 3 real matrices.

(b) Matrices with distinct eigenvalues are dense in the set of 3 × 3 complex matrices.

Solution.

(a) Matrices with determinant 1 are not dense in the set of 3 × 3 real matrices. The
determinant map is a C ∞ -smooth map. Thus if the determinant was 1 on a dense set,
then every matrix would have determinant 1.

(b) Matrices with distinct eigenvalues are dense in the set of 3 × 3 complex matrices.
We prove this for upper triangular matrices first. Let ε > 0 and let T be an upper
triangular 3 × 3 matrix. We show there is a matrix with distinct eigenvalues within ε

48
of T . If T already has distinct eigenvalues, we’re done. Otherwise, there are two cases
we consider:
Case 1: All diagonal
√ √entries
√ are T are the same value t. In this case, let
Dε = diag(ε/ 4, ε/ 6, ε/ 12). Then T + Dε has distinct eigenvalues and
p p √
||T − (T + Dε )|| = ||Dε || = ε2 /4 + ε2 /6 + ε2 /12 = ε2 /2 = ε/ 2 < ε.

Case 2: Two of the diagonal entries of T are the same while the other is different.
Suppose without loss of generality that the first two are the same while the third is
√ eigenvalues a and b. Let δ = |b − a| . Set Dε = diag(t, −t, t) where
different. Call the
t < min{δ/3, ε/ 5}. Then T + Dε has distinct eigenvalues and
p p
||T − (T + Dε )|| = ||Dε || < 3ε2 /5 = ε 3/5 < ε.

Thus the claim holds for upper triangular matrices. However, every matrix is similar to
an upper triangular matrix by the Schur decomposition (see W02.11). Thus adding
to the diagonal of the original matrix changes the eigenvalues the in the same way
as adding to the diagonal of upper triangular matrix to which the original is similar.
Hence the claim holds for all matrices.

Problem S15.9. Let V = Rn and let U1 , U2 , W1 , W2 be subspaces of V of dimension d such


that dim(U1 ∩ W1 ) = dim(U2 ∩ W2 ) = `, ` ≤ d ≤ n. Prove that there is a linear operator
T : V → V such that T (U1 ) = U2 and T (W1 ) = W2 .

Solution. Not sure.

Problem 15.10. Let


   
A B −1 P Q
M= and M =
C D R S

where A, B, C, D, P, Q, R, S are all k × k matrices. Show that

det(M ) · det(S) = det(A).

Solution. Not sure.

Problem S15.11. Two matrices A, B are called commuting if AB = BA. The order of a
matrix A is defined to be the smallest non-negative integer k such that Ak = I; if no such k
exists, the matrix is said to have infinite order. Prove that there exist ten distinct real 2 × 2
matrices which are pairwise commuting and have the same finite order.

Solution. Matrices of the form  


cos θ − sin θ
sin θ cos θ

49
always commute and have the property that
 n  
cos θ − sin θ cos(nθ) − sin(nθ)
= .
sin θ cos θ sin(nθ) cos(nθ)

Choosing θ = 2πk/11, k = 1, 2, . . . , 11 gives ten different matrices, all of order 11 which


pairwise commute.
Note: the reason you should think of these matrices is because, together with the identity,
they form a group which is isomorphic to the subgroup of D11 (the symmetries of the regular
11-gon) consisting of rotations.

Problem S15.12. Let  


3 5
M= .
1 −1

(a) Compute exp(M ).

(b) Is there a real matrix A such that M = exp(A)?

Solution.

(a) To compute exp(M ) we first find a Jordan form for M . The characteristic polynomial
of M is
pM (t) = (3 − t)(−1 − t) − 5 = t2 − 2t − 8 = (t − 4)(t + 2).
Thus M has distinct eigenvalues and so it is diagonalizable. An eigenvector correspond-
ing to λ1 = 4 is given by v1 = (5, 1)t and an eigenvector corresponding to λ2 = −2 is
v2 = (1, −1)t . Putting P = [v1 v2 ], we see
   
−1 1 −1 −1 3 5 5 1
P MP = −
6 −1 5 1 −1 1 −1
  
1 −1 −1 20 −2
=−
6 −1 5 4 2
   
1 −24 0 4 0
=− = .
6 0 12 0 −2.

So M = P DP −1 where D = diag(4, −2). Then


  4  
−1 1 5 1 e 0 −1 −1
exp(M ) = P exp(D)P = −
6 1 −1 0 e−2 −1 5
  4 4

1 5 1 −e −e
=−
6 1 −1 −e−2 5e−2
1 −5e4 − e−2 −5e4 + 5e−2 1 5e4 + e−2 5e4 − 5e−2
   
=− = .
6 −e4 + e−2 −e4 − 5e−2 6 e4 − e−2 e4 + 5e−2

(b) No. The eigenvalues of exp(A) are eλ1 , eλ2 where λ1 , λ2 are the eigenvalues of A.

50
Suppose exp(A) = M . If A has real eigenvalues λ1 , λ2 , then (wlog) eλ1 = 4, eλ2 = −2,
but this is impossible for λ2 ∈ R.
If A has complex eigenvalues then they must be a conjugate pair: λ, λ. But then eλ
and eλ form a conjugate pair which is impossible since eλ = 4, eλ = −2 (or vice versa).
Problem F15.7. Let A, B be two 4 × 5 matrices of rank 3. Find all possible values for
the rank of C = At B. Specifically, you must find examples for any values possible and prove
that no other values are possible.

Solution. The possible values for rank(C) are 2 and 3. Putting


   
1 0 0 0 0 1 0 0 0 0
0 1 0 0 0 0 0 0 0 0
A= 0
, B =  
0 1 0 0 0 0 0 1 0
0 0 0 0 0 0 0 0 0 1

gives rank(C) = 2. Putting


   
1 0 0 0 0 1 0 0 0 0
0 1 0 0 0 0 1 0 0 0
A=
0
, B= 
0 1 0 0 0 0 1 0 0
0 0 0 0 0 0 0 0 0 0

gives rank(C) = 3.
Let {v1 , v2 , v3 } ⊂ C4 be a basis for im(B) (since rank(B) = 3). Take y ∈ im(C). Then
there is v ∈ C5 such that Cv = y. Then At (Bv) = y. But Bv ∈ im(B) so there are
α1 , α2 , α3 ∈ C such that Bv = α1 v1 + α2 v2 + α3 v3 . Then

y = α1 At v1 + α2 At v2 + α3 At v3 .

Since y ∈ im(C) was arbitrary, this shows that {At v1 , At v2 , At v3 } is a spanning set for
im(C). Thus rank(C) ≤ 3 since the rank is less than or equal to the number of elements in
any spanning set.
Consider, since rank(A) = 3, we have rank(At ) = 3. Then by the rank-nullity theorem,
dim(ker(At )) = 1 since At acts on C4 . Then since v1 , v2 , v3 are linearly independent, there is
at most one i = 1, 2, 3 such that At vi = 0. Suppose there is one; wlog, At v3 = 0. Then {v3 }
must form a basis for ker(At ) since the kernel has dimension 1. Let β1 , β2 ∈ C be such that

β1 At v1 + β2 At v2 = 0.

Then At (β1 v1 + β2 v2 ) = 0 so β1 v1 + β2 v2 ∈ ker(At ). Then there is β3 ∈ C such that

β1 v1 + β2 v2 = β3 v3 .

But these vectors form a basis for im(B) so this implies (in particular) that β1 = β2 = 0.
Thus At v1 and At v2 are linearly independent. However, they are in im(C) so this implies
rank(C) ≥ 2 since the rank is greater than or equal to the number of elements in any linearly
independent subset.

51
The case where there is no i = 1, 2, 3 such that Avi = 0 is similar.

Problem F15.8. Find M −2 where


 
2 3 2 1
3 6 4 2
M =
4
.
8 6 3
2 4 3 1

Solution. I’m not sure there is a “clever” way to do this. Just perform elementary row
operations on M until you have the identity and then perform those same row operations to
the identity to find M −1 . Doing this, we find
 
2 −1 0 0
−1 2 −1 0 
M −1 =  0 −2 1
.
1
0 0 1 −2

Then squaring, we get


    
2 −1 0 0 2 −1 0 0 5 −4 1 0
−1 2 −1 0  −1 2 −1 0  −4 7 −3 −1
M −2 = 
 0 −2 1
 = .
1   0 −2 1 1   2 −6 4 −1
0 0 1 −2 0 0 1 −2 0 −2 −1 5

Problem F15.9. Let A be an n×n real matrix such that At = −A. Prove that det(A) ≥ 0.

Solution. If A is not invertible, then det(A) = 0 so the claim is trivially satisfied.


Suppose A is invertible. Then A does not have zero as an eigenvalue. Let λ ∈ C − {0}
be an eigenvalue of A with corresponsing eigenvector 0 6= v ∈ Cn . Then

−λ(v, v) = (−λv, v) = (−Av, v) = (At v, v) = (v, Av) = (v, λv) = λ(v, v).

Then since (v, v) > 0, we have −λ = λ. But this yields Re(λ) = 0. Thus all eigenvalues of
A are purely imaginary. Also the non-real eigenvalues of A come in conjugate pairs since
A is real. Thus the eigenvalues of A can be listed: iµ1 , −iµ1 , . . . , iµ` , −iµ` for some ` ∈ N,
µ1 , . . . , µ` ∈ R − {0}. The determinant of A is the product of the eigenvalues so

det(A) = µ21 · · · µ2` ≥ 0.

[Note: interestingly enough, this actually shows that if n is odd, then 0 must be an
eigenvalue of A. Thus if A is an n × n skew-symmetric invertible matrix, then n is even.]

Problem F15.10. Let F, G : Rn → Rn be linear operators. Recall, we define the operator


exponential by

X 1 k
exp(F ) = F .
k=0
k!

52
(a) Prove that when F and G commute, we have

exp(F + G) = exp(F ) exp(G).

(b) Find two non-commuting linear operators such that this equality fails.

Solution.

(a) If F, G commute, then the binomial theorem holds for F, G. That is,
k  
k
X k
(F + G) = F ` Gk−` , k = 0, 1, . . .
`=0
`

where by definition, F 0 = I = G0 where I is the identity operator on Rn . Recall the


Cauchy product of two infinite series:

! ∞ ! ∞ X k
X X X
ak b` = a` bk−`
k=0 `=0 k=0 `=0

when all series converge absolutely. Using these we have



! ∞ !
X 1 k X1
exp(F ) exp(G) = F G`
k=0
k! `=0
`!
∞ X k   
X 1 ` 1 k−`
= F G
k=0 `=0
`! (k − `)!
∞ k  
X 1 X k
= F ` Gk−`
k=0
k! `=0 `

X 1
= (F + G)k = exp(F + G).
k=0
k!

(b) Let    
1 1 1 0
A= , B=
0 1 1 1
We see       
0 1 e 0 1 1 e e
exp(A) = exp(I) exp = =
0 0 0 e 0 1 0 e
and likewise  
e 0
exp(B) =
e e
so     2 2
e e e 0 2e e
exp(A) exp(B) = = .
0 e e e e2 e2

53
However,  
2 1
exp(A + B) = exp = exp(2I) exp(J),
1 2
 
0 1
where J = so J 2 = I. Then
1 0
∞ ∞  
X 1 X 1 cosh(1) sinh(1)
exp(J) = I+ J=
(2k)! (2k + 1)! sinh(1) cosh(1)
k=0 k=0

so  2 
e cosh(1) e2 sinh(1)
exp(A + B) = 6= exp(A) exp(B).
e2 sinh(1) e2 cosh(1)

Problem F15.11. Let T : V → V be a linear operator such that T 6 = 0 and T 5 6= 0.


Suppose V ' R6 . Prove there is no linear operator S : V → V such that S 2 = T . Does the
answer change if V ' R12 ?

Solution. Suppose that V ' R6 and that there is a linear operator S : V → V such that
T = S 2 . Then 0 = T 6 = S 12 and 0 6= T 5 = S 10 . Let λ ∈ C be an eigenvalue of S. Then
λ12 is an eigenvalue of S 12 = 0 so λ12 = 0 and so λ = 0. Thus all eigenvalues of S are zero.
Then the characteristic polynomial of S is pS (x) = x6 since S acts on a 6 dimensional space.
However, the Cayley-Hamilton theorem states that pS (S) = 0 so S 6 = 0 =⇒ S 10 = 0; a
contradiction. Thus no such S exists.
Yes, the answer does change if V is 12 dimensional. Let V = R12 and T, S be the matrices

0 1 0 0 ··· 0 0
   
0 0 1 0 ··· 0 0
0 0 0 1 · · · 0 0  0 0 1 0 · · · 0 0 
. . .. 
..
 
 .. .. .. .. .. . . .
. . .
 
. . . . .  
T = 0 0 0 0 · · · 1 0 , S =  ... ... ... ..  ;
  
   .
0 0 0 0 · · · 0 1  0 0 0 0 · · · 1 0 
   
0 0 0 0 · · · 0 0  0 0 0 0 · · · 0 1 
0 0 0 0 ··· 0 0 0 0 0 0 ··· 0 0

that is T has all zeroes except ones on the second superdiagonal and S is all zeroes except
ones on the first superdiagonal. Then T 5 6= 0, T 6 = 0 and S 2 = T .

Problem F15.12. Prove that the n × n matrix M is positive definite:


 
2 1 1 ··· 1
1 3 1 · · · 1 
 
M = 1 1 4 · · · 1 .

 .. .. .. . . .. 
. . . . . 
1 1 1 ··· n + 1

54
Solution. Let x = (x1 , . . . , xn ) ∈ Cn and let M = (mij )1≤i,j≤n . Then
n
X
x∗ M x = mij xi xj
i,j=1

= 2 |x1 |2 + x1 x2 + x1 x3 + · · · + x1 xn−1 + x1 xn
+ x2 x1 + 3 |x2 |2 + x2 x3 + · · · + x2 xn−1 + x2 xn
..
.
+ xn x1 + xn x2 + xn x3 + · · · + xn xn−1 + (n + 1) |xn |2 .

From here, we see

|xi |2 + xi xi+1 + xi+1 xi + |xi+1 |2 = (xi + xi+1 )(xi + xi+1 ) = (xi + xi+1 )(xi + xi+1 ) = |xi + xi+1 |2

for each i = 1, . . . , n − 1, so

x∗ M x = |x1 |2 + |x2 |2 + 2 |x3 |2 + · · · + (n − 2) |xn−1 |2 + n |xn |2


+ |x1 + x2 |2 + |x2 + x3 |2 + · · · + |xn−1 + xn |2

from which it is clear that x∗ M x ≥ 0 and so M is positive definite.


[Note: since M is real, it actually suffices to check x ∈ Rn . If v ∈ Cn , then v =
x + iy, x, y ∈ Rn and v ∗ M v = xt M x + y t M y. Thus proving that v ∗ M v ≥ 0 is reduced to
showing that xt M x, y t M y ≥ 0.]

55

S-ar putea să vă placă și